You are on page 1of 64

ENARM 11ª PARTE by JD-MD

ENARM by JD-MD 11a PARTE

1001.- un efecto adverso que puede ocurrir si se administra hormona del


crecimiento antes de la pubertad es:

Cierre prematuro de las metáfisis


Hiperlipoproteinemia
Aparición de diabetes
Gigantismo
Acromegalia

Excessive GH causes tall stature and gigantism if it occurs before closure of epiphyses.
Afterward, acromegaly develops. The term "acromegaly," meaning extremity
enlargement, seriously understates the manifestations. The hands enlarge and a
doughy, moist handshake is characteristic. The fingers widen, causing patients to
enlarge their rings. Carpal tunnel syndrome is common. The feet also grow, particularly
in width. Facial features coarsen since the bones and sinuses of the skull enlarge; hat
size increases. The mandible becomes more prominent, causing prognathism and
malocclusion. Tooth spacing widens. Macroglossia occurs, as does hypertrophy of
pharyngeal and laryngeal tissue; this causes a deep, coarse voice and sometimes
makes intubation difficult. Obstructive sleep apnea may occur. A goiter may be noted.
Hypertension (50%) and cardiomegaly are common. At diagnosis, about 10% of
acromegalic patients have overt heart failure, with a dilated left ventricle and a
reduced ejection fraction. Weight gain is typical, particularly of muscle and bone.
Insulin resistance is usually present and frequently causes diabetes mellitus (30%).
Arthralgias and degenerative arthritis occur. Overgrowth of vertebral bone can cause
spinal stenosis. Colon polyps are common, especially in patients with skin papillomas.
The skin may also manifest hyperhidrosis, thickening, cystic acne, and areas of
acanthosis nigricans. GH-secreting pituitary tumors usually cause some degree of
hypogonadism, either by cosecretion of prolactin or by direct pressure upon normal
pituitary tissue. Decreased libido and impotence are common, as are irregular menses
or amenorrhea. Secondary hypothyroidism sometimes occurs; hypoadrenalism is
unusual. Headaches are frequent. Temporal hemianopia may occur as a result of the
optic chiasm being impinged by a suprasellar growth of the tumor. Endoscopic
transnasal, transsphenoidal pituitary microsurgery removes the adenoma while
preserving anterior pituitary function in most patients. Surgical remission is achieved in
about 70% of patients followed over 3 years. GH levels fall immediately; diaphoresis
and carpal tunnel syndrome often improve within a day after surgery. Transsphenoidal
surgery is usually well tolerated, but complications occur in about 10% of patients,
including infection, cerebrospinal fluid leak, and hypopituitarism. Hyponatremia can
occur 4-13 days postoperatively and is manifested by nausea, vomiting, headache,
malaise, or seizure. It is prudent to monitor serum sodium levels postoperatively.
Dietary salt supplements for 2 weeks postoperatively may prevent this complication.
Patients who fail to have a clinical or biochemical remission after surgery are treated
with a dopamine agonist (eg, cabergoline), octreotide, pegvisomant, or a combination
of these medications. Cabergoline may be used first, since it is an oral medication.
Cabergoline therapy is most successful for tumors that secrete both prolactin (PRL) and
GH, but can also be effective for patients with normal serum PRL levels. Therapy with
cabergoline will shrink one-third of such tumors by more than 50%. Cabergoline is
administered orally, starting with 0.25 mg twice weekly. If the patient tolerates
cabergoline, the dosage may be increased gradually, based upon serum GH and IGF-I
levels; the maximum dosage is 1 mg orally twice weekly. Side effects of cabergoline
include nausea, fatigue, constipation, abdominal pain, and dizziness. Cabergoline is
expensive. Somatostatin analogs may be used to treat patients who have persistent
acromegaly despite pituitary surgery. Octreotide and lanreotide are somatostatin

MX-Biomedical Research Group / JD-MD Bioinformatics Labs 1


JD-MD Medicine and Technology
ENARM 11ª PARTE by JD-MD

analogs that are given by subcutaneous injection. Short-acting octreotide acetate in


doses of 50 mcg is injected subcutaneously three times daily. Responders who tolerate
the drug are switched to long-acting octreotide acetate injectable suspension in a
dosage of 20 mg intragluteally per month. The dosage may be adjusted¾up to a
maximum of 40 mg monthly¾to maintain the serum GH between 1 and 2.5 ng/mL,
keeping IGF-I levels normal. Lanreotide SR (not available in the United States) is given
by subcutaneous injection at a dosage of 30 mg every 7-14 days. Lanreotide Autogel
(not available in the United States) is a newer formulation that is administered by deep
subcutaneous injection in doses of 60-120 mg every 28 days; this preparation is better
tolerated than lanreotide SR. All somatostatin analogs are expensive and must be
continued indefinitely or until other treatment has been effective. Octreotide long-
acting release (LAR) preparations (Sandostatin LAR depot) are superior to shorter-
acting octreotide, ultimately achieving serum GH levels under 2 ng/mL in 79% of
patients and normal serum IGF-I levels in 53% of patients. Headaches often improve,
and tumor shrinkage of about 30% may be expected. Acromegalic patients with
pretreatment serum GH levels exceeding 20 ng/mL are less likely to respond to
octreotide therapy. Side effects are experienced by about one-third of patients and
include injection site pain, loose acholic stools, abdominal discomfort, or cholelithiasis.
Pegvisomant is a GH receptor antagonist that blocks the effects of GH. Pegvisomant
therapy produces symptomatic relief and normalizes serum IGF-I levels in over 90% of
acromegalic patients. The starting dosage is 10 mg subcutaneously daily. The
maintenance dosage can be increased by 5-10 mg every 4-6 weeks, based upon serum
IGF-I levels and liver transaminase levels; the maximum dosage is 30 mg
subcutaneously daily. Pegvisomant does not shrink GH-secreting tumors. Patients need
to be monitored carefully with visual field examinations, GH levels, and MRI scanning of
the pituitary. Side effects of pegvisomant can include injection site reactions, hepatitis,
edema, flu-like syndrome, nausea, and hypertension. In acromegalic diabetics,
hypoglycemic drugs are reduced to avoid hypoglycemia during pegvisomant therapy.
The effectiveness of pegvisomant is reduced by coadministration of opioids or
propoxyphene. Pegvisomant is extraordinarily expensive. Acromegalic patients who
have not had a complete remission with transsphenoidal surgery or medical therapy
may be treated with stereotactic radiosurgery administered by gamma knife, heavy
particle radiation, or adapted linear accelerator. Radiosurgery precisely radiates the
pituitary tumor in a single session and reduces radiation to the normal brain. However,
it cannot be used for pituitary tumors with suprasellar extension due to the risk of
damaging the optic chiasm. Radiosurgery can be used for pituitary tumors invading the
cavernous sinus, since cranial nerves III, IV, V, and VI are less susceptible to radiation
damage. Compared to conventional radiation therapy, radiosurgery is more effective
and carries a lower risk of hypopituitarism, cerebral necrosis, psychological
impairment, and small vessel stroke. Radiosurgery can also be used for patients who
have failed conventional radiation therapy.

1002.- Tipo de enfisema que se asocia al tabaquismo:

a) Paraseptal
b) Irregular
c) Panlobulillar
d) Centrolobulillar
e) Parahiliar

El enfisema es caracterizado por el aumento permanente y anómalo de los espacios


aéreos distales al bronquiolo terminal, acompañado de destrucción de sus paredes, y
sin signos de fibrosis. Existen 4 tipos principales: 1) centroacinar (centrolobulillar); 2)
panacinar; 3) paraseptal, y 4) irregular. Solo los dos primeros causan una obstrucción
del flujo de aire clínicamente significativa. El enfisema centroacinar es mucho más

MX-Biomedical Research Group / JD-MD Bioinformatics Labs 2


JD-MD Medicine and Technology
ENARM 11ª PARTE by JD-MD

frecuente que la forma panacinar y representa el 95% de los casos. Los productos del
tabaco y el carbón tienen un papel predominante en la génesis de este tipo de
enfisema.

1003.- El control mensual rutinario de la tuberculosis pulmonar se debe de


hacer por medio de:

a) Cultivo de exudado
b) Pruebas de sensibilidad
c) Microscopia de esputo
d) Tele de tórax
e) Examen clínico

En el control del tratamiento antituberculosis, con una muestra mensual de


expectoración con apoyo de los métodos convencionales (puño y palmo-percusión),
toda muestra de control independientemente de la calidad y apariencia macroscópica
debe ser procesada, principalmente al final del tratamiento.

1003.- Principal procedimiento diagnóstico para la detección de


tromboembolia pulmonar:

a) Broncografía contrastada
b) Broncoscopía directa
c) Gammagrafía de perfusión pulmonar
d) Tele de tórax en posición supina
e) Espirometría computarizada
De las opciones presentadas la que parece “mas verdadera es la opción C”, enseguida
se cita una breve referencia.
Pulmonary arteriography remains the reference standard for the diagnosis of
pulmonary thromboembolism. An intraluminal filling defect in more than one projection
establishes a definitive diagnosis. Secondary findings highly suggestive of pulmonary
thromboembolism include abrupt arterial cutoff, asymmetry of blood flow¾especially
segmental oligemia¾or a prolonged arterial phase with slow filling. Pulmonary
arteriography was performed in 755 patients in the PIOPED study. A definitive diagnosis
was established in 97%; in 3% the studies were nondiagnostic. Four patients (0.8%)
with negative arteriograms subsequently had pulmonary thromboemboli at autopsy.
Serial arteriography has demonstrated minimal resolution of thrombus prior to day 7
following presentation. Thus, negative arteriography within 7 days of presentation
excludes the diagnosis. Pulmonary arteriography is a safe but invasive procedure with
well-defined morbidity and mortality data. Minor complications occur in approximately
5% of patients. Most are allergic contrast reactions, transient renal dysfunction, or
related to percutaneous catheter insertion; cardiac perforation and arrhythmias are
reported but rare. Among the PIOPED patients who underwent arteriography, there
were five deaths (0.7%) directly related to the procedure. Pulmonary hypertension is
thought to increase the risk of serious complications, though a study of patients with
average pulmonary arterial pressures of 74/34 mm Hg developed no major
complications or deaths associated with pulmonary arteriography. The appropriate role
of pulmonary arteriography in the diagnosis of pulmonary thromboembolism remains a
subject of active debate. There is wide agreement that arteriography is indicated in
several specific situations: in patients with nondiagnostic V/Q scans, intermediate or
high clinical pretest probability of pulmonary thromboembolism, and negative
noninvasive leg studies; in any patient in whom the diagnosis is in doubt when there is
a high clinical pretest probability of pulmonary thromboembolism; and when the
diagnosis of pulmonary thromboembolism must be established with certainty, as when
MX-Biomedical Research Group / JD-MD Bioinformatics Labs 3
JD-MD Medicine and Technology
ENARM 11ª PARTE by JD-MD

anticoagulation is contraindicated or placement of an inferior vena cava filter is


contemplated. A perfusion scan is performed by injecting radiolabeled
microaggregated albumin into the venous system, allowing the particles to embolize to
the pulmonary capillary bed. To perform a ventilation scan, the patient breathes a
radioactive gas or aerosol while the distribution of radioactivity in the lungs is recorded.
A defect on perfusion scanning represents diminished blood flow to that region of the
lung. This finding is not specific for pulmonary embolism. Defects in the perfusion scan
are interpreted in conjunction with the ventilation scan to give a high, low, or
intermediate (indeterminate) probability that pulmonary thromboembolism is the cause
of the abnormalities. Criteria for the combined interpretation of ventilation and
perfusion scans (commonly referred to as a single test, the V/Q scan) are complex,
confusing, and not completely standardized. A normal perfusion scan excludes the
diagnosis of clinically significant pulmonary thromboembolism (negative predictive
value of 91% in the PIOPED study). A high-probability V/Q scan is most often defined as
having two or more segmental perfusion defects in the presence of normal ventilation
and is sufficient to make the diagnosis of pulmonary thromboembolism in most
instances (positive predictive value of 88% among PIOPED patients). In the presence of
abnormal pulmonary vasculature, as commonly happens in prior pulmonary
thromboembolism, or if the clinical pretest probability for embolism is low, angiography
may be indicated even in the presence of a high-probability V/Q scan. Helical CT
arteriography is rapidly supplanting V/Q scanning as the initial diagnostic study for
suspected pulmonary thromboembolism. Helical CT arteriography requires
administration of intravenous radiocontrast dye but is otherwise noninvasive. It is very
sensitive for the detection of thrombus in the proximal pulmonary arteries but less so
in the segmental and subsegmental arteries. Test results vary widely by study and
facility. Factors influencing results include patient size and cooperation, the type and
quality of the scanner, the imaging protocol, and the experience of the radiologist. One
report comparing helical CT with standard arteriography reported sensitivity of 53-60%
and specificity of 81-97%. Comparing helical CT to the V/Q scan as the initial test for
pulmonary thromboembolism, detection of thrombi is comparable, but more
nonthromboembolism pulmonary diagnoses are made with CT scanning. Independent
of cost and availability, helical CT may offer advantages as a screening examination,
especially in hospitalized patients and in patients with significant comorbidities. A
contentious issue is whether a negative helical CT requires any further evaluation.
False-negative results may occur in up to 20% of helical CTs. Advocates contend that
these false-negatives represent small peripheral thromboemboli and that such patients
can be monitored off anticoagulation without undue risk. One study reported a venous
thromboembolism rate of 0.8% in 3-month follow-up of 376 patients with negative
helical CT scans, but the mortality rate at 3 months was 10.1%. Further study is
required to clarify the role of this diagnostic modality, especially in view of ongoing
advances in CT technology and the increasing availability of multi-detector-row
scanners.

1004.- Síntoma más frecuente de la tromboembolia pulmonar aguda:

a) Hemoptisis
b) Tos
c) Síncope
d) Disnea
e) Dolor torácico pleurítico

The clinical diagnosis of pulmonary thromboembolism is notoriously difficult for two


reasons. First, the clinical findings depend on both the size of the embolus and the
patient's preexisting cardiopulmonary status. Second, common symptoms and signs of
pulmonary emboli are not specific to this disorder. Indeed, no single symptom or sign

MX-Biomedical Research Group / JD-MD Bioinformatics Labs 4


JD-MD Medicine and Technology
ENARM 11ª PARTE by JD-MD

or combination of clinical findings is specific to pulmonary thromboembolism. Some


findings are fairly sensitive: dyspnea and pain on inspiration occur in 75-85%
and 65-75% of patients, respectively. Tachypnea is the only sign reliably
found in more than half of patients. A common clinical strategy is to use
combinations of clinical findings to identify patients at low risk for pulmonary
thromboembolism. For example, 97% of patients in the Prospective Investigation of
Pulmonary Embolism Diagnosis (PIOPED) study with angiographically proved pulmonary
emboli had one or more of three findings: dyspnea, chest pain with breathing, or
tachypnea. Such a sensitive screen allows exclusion of the diagnosis on clinical
grounds in a small number of patients. To establish the diagnosis or to exclude it
definitively, further testing is required in the majority of patients.

1005.- Signo más frecuente en la tromboembolia pulmonar aguda:

a) Fiebre
b) Flebitis
c) Cianosis
d) Taquipnea
e) Taquicardia

The clinical diagnosis of pulmonary thromboembolism is notoriously difficult for two


reasons. First, the clinical findings depend on both the size of the embolus and the
patient's preexisting cardiopulmonary status. Second, common symptoms and signs of
pulmonary emboli are not specific to this disorder. Indeed, no single symptom or sign
or combination of clinical findings is specific to pulmonary thromboembolism. Some
findings are fairly sensitive: dyspnea and pain on inspiration occur in 75-85%
and 65-75% of patients, respectively. Tachypnea is the only sign reliably
found in more than half of patients. A common clinical strategy is to use
combinations of clinical findings to identify patients at low risk for pulmonary
thromboembolism. For example, 97% of patients in the Prospective Investigation of
Pulmonary Embolism Diagnosis (PIOPED) study with angiographically proved pulmonary
emboli had one or more of three findings: dyspnea, chest pain with breathing, or
tachypnea. Such a sensitive screen allows exclusion of the diagnosis on clinical
grounds in a small number of patients. To establish the diagnosis or to exclude it
definitively, further testing is required in the majority of patients.

1006.- Característica física de la acromegalia:

a) Micrognatia
b) Prognatismo
c) Disminución del tamaño de los senos paranasales
d) Hipotelorismo
e) Microglosia

Excessive GH causes tall stature and gigantism if it occurs before closure of epiphyses.
Afterward, acromegaly develops. The term "acromegaly," meaning extremity
enlargement, seriously understates the manifestations. The hands enlarge and a
doughy, moist handshake is characteristic. The fingers widen, causing patients to
enlarge their rings. Carpal tunnel syndrome is common. The feet also grow, particularly
in width. Facial features coarsen since the bones and sinuses of the skull enlarge; hat
size increases. The mandible becomes more prominent, causing prognathism
and malocclusion. Tooth spacing widens. Macroglossia occurs, as does
hypertrophy of pharyngeal and laryngeal tissue; this causes a deep, coarse voice and
sometimes makes intubation difficult. Obstructive sleep apnea may occur. A goiter may
be noted. Hypertension (50%) and cardiomegaly are common. At diagnosis, about 10%
MX-Biomedical Research Group / JD-MD Bioinformatics Labs 5
JD-MD Medicine and Technology
ENARM 11ª PARTE by JD-MD

of acromegalic patients have overt heart failure, with a dilated left ventricle and a
reduced ejection fraction. Weight gain is typical, particularly of muscle and bone.
Insulin resistance is usually present and frequently causes diabetes mellitus (30%).
Arthralgias and degenerative arthritis occur. Overgrowth of vertebral bone can cause
spinal stenosis. Colon polyps are common, especially in patients with skin papillomas.
The skin may also manifest hyperhidrosis, thickening, cystic acne, and areas of
acanthosis nigricans. GH-secreting pituitary tumors usually cause some degree of
hypogonadism, either by cosecretion of prolactin or by direct pressure upon normal
pituitary tissue. Decreased libido and impotence are common, as are irregular menses
or amenorrhea. Secondary hypothyroidism sometimes occurs; hypoadrenalism is
unusual. Headaches are frequent. Temporal hemianopia may occur as a result of the
optic chiasm being impinged by a suprasellar growth of the tumor.

1007.- Patrón menstrual más común en el hipotiroidismo:

a) Hipermenorrea
b) Menorragia
c) Oligomenorrea
d) Metrorragia
e) Amenorrea

ESSENTIALS OF DIAGNOSIS OF HYPOTHYROIDISM AND MIXEDEMA:


• Weakness, fatigue, cold intolerance, constipation, weight change, depression,
menorrhagia, hoarseness
• Dry skin, bradycardia, delayed return of deep tendon reflexes
• Anemia, hyponatremia
• T4 and RAI uptake usually low
• TSH elevated in primary hypothyroidism
1008.- Paciente femenino de 25 años de edad, diagnóstico de anemia
establecido, hay antecedentes familiares de cálculos biliares a edad
temprana en muchos miembros de su familia. Esta combinación sugiere el
diagnóstico de:

a) Deficiencia de vitamina B 12
b) Rasgo de talasemia alfa
c) Deficiencia de hierro
d) Esferocitosis hereditaria
e) Hemoglobinuria nocturna paroxística

Hereditary spherocytosis is an autosomal dominant disease of variable severity. It is


often diagnosed during childhood, but milder cases may be discovered incidentally late
in adult life. Anemia may or may not be present, since the bone marrow may be able to
compensate for shortened red cell survival. Severe anemia (aplastic crisis) may occur
in folic acid deficiency or when bone marrow compensation is temporarily impaired by
infection. Chronic hemolysis causes jaundice and pigment (calcium bilirubinate)
gallstones, leading to attacks of cholecystitis. Examination may reveal icterus and a
palpable spleen.

1009.- Mejor prueba de detección para distinguir entre la anemia ferropénica


y la anemia por enfermedad crónica en un paciente que acude a la consulta
externa para valoración de un síntoma de fatiga y se detecta anemia
microcítica:

MX-Biomedical Research Group / JD-MD Bioinformatics Labs 6


JD-MD Medicine and Technology
ENARM 11ª PARTE by JD-MD

a) Ferritina sérica
b) Cuenta de reticulocitos
c) Hierro sérico
d) Porcentaje de saturación de transferrina
e) Reservas de hierro de la medula ósea

Iron deficiency is the most common cause of anemia worldwide. Iron is necessary for
the formation of heme and other enzymes. Total body iron ranges between 2 and 4 g:
approximately 50 mg/kg in men and 35 mg/kg in women. Most (70-95%) of the iron is
present in hemoglobin in circulating red blood cells. One milliliter of packed red blood
cells (not whole blood) contains approximately 1 mg of iron. In men, red blood cell
volume is approximately 30 mL/kg. A 70-kg man will therefore have approximately
2100 mL of packed red blood cells and consequently 2100 mg of iron in his circulating
blood. In women, the red cell volume is about 27 mL/kg; a 50-kg woman will thus have
1350 mg of iron circulating in her red blood cells. Only 200-400 mg of iron is present in
myoglobin and nonheme enzymes. Aside from circulating red blood cells, the major
location of iron in the body is the storage pool, as ferritin or as hemosiderin and in
macrophages. The range for storage iron is wide (0.5-2 g); approximately 25% of
women in the United States have none. The average American diet contains 10-15 mg
of iron per day. About 10% of this amount is absorbed. Absorption occurs in the
stomach, duodenum, and upper jejunum. Dietary iron present as heme is efficiently
absorbed (10-20%) but nonheme iron less so (1-5%), largely because of interference by
phosphates, tannins, and other food constituents. Small amounts of
iron¾approximately 1 mg/d¾are normally lost though exfoliation of skin and mucosal
cells. There is no physiologic mechanism for increasing normal body iron losses.
Menstrual blood loss in women plays a major role in iron metabolism. The average
monthly menstrual blood loss is approximately 50 mL, or about 0.7 mg/d. However,
menstrual blood loss may be five times the average. To maintain adequate iron stores,
women with heavy menstrual losses must absorb 3-4 mg of iron from the diet each
day. This strains the upper limit of what may reasonably be absorbed, and women with
menorrhagia of this degree will almost always become iron deficient without iron
supplementation. In general, iron metabolism is balanced between absorption of 1
mg/d and loss of 1 mg/d. Pregnancy may also upset the iron balance, since
requirements increase to 2-5 mg of iron per day during pregnancy and lactation.
Normal dietary iron cannot supply these requirements, and medicinal iron is needed
during pregnancy and lactation. Repeated pregnancy (especially with breast-feeding)
may cause iron deficiency if increased requirements are not met with supplemental
medicinal iron. Decreased iron absorption can on very rare occasions cause iron
deficiency and usually occurs after gastric surgery, though concomitant bleeding is
frequent. By far the most important cause of iron deficiency anemia is blood loss,
especially gastrointestinal blood loss. Chronic aspirin use may cause it even without a
documented structural lesion. Iron deficiency demands a search for a source of
gastrointestinal bleeding if other sites of blood loss (menorrhagia, other uterine
bleeding, and repeated blood donations) are excluded. Chronic hemoglobinuria may
lead to iron deficiency since iron is lost in the urine; traumatic hemolysis due to a
prosthetic cardiac valve and other causes of intravascular hemolysis (eg, paroxysmal
nocturnal hemoglobinuria) should also be considered. As a rule, the only symptoms of
iron deficiency anemia are those of the anemia itself (easy fatigability, tachycardia,
palpitations and tachypnea on exertion). Severe deficiency causes skin and mucosal
changes, including a smooth tongue, brittle nails, and cheilosis. Dysphagia because of
the formation of esophageal webs (Plummer-Vinson syndrome) also occurs. Many iron-
deficient patients develop pica, craving for specific foods (ice chips, etc) often not rich
in iron. Iron deficiency develops in stages. The first is depletion of iron stores. At this
point, there is anemia and no change in red blood cell size. The serum ferritin will
become abnormally low. A ferritin value less than 30 mcg/L is a highly reliable indicator
MX-Biomedical Research Group / JD-MD Bioinformatics Labs 7
JD-MD Medicine and Technology
ENARM 11ª PARTE by JD-MD

of iron deficiency. The serum total iron-binding capacity (TIBC) rises. Bone marrow
biopsy for evaluation of iron stores is now rarely performed because of intraobserver
variation in its interpretation. After iron stores have been depleted, red blood cell
formation will continue with deficient supplies of iron. Serum iron values decline to less
than 30 mcg/dL and transferrin saturation to less than 15%. In the early stages, the
MCV remains normal. Subsequently, the MCV falls and the blood smear shows
hypochromic microcytic cells. With further progression, anisocytosis (variations in red
blood cell size) and poikilocytosis (variation in shape of red cells) develop. Severe iron
deficiency will produce a bizarre peripheral blood smear, with severely hypochromic
cells, target cells, hypochromic pencil-shaped cells, and occasionally small numbers of
nucleated red blood cells. The platelet count is commonly increased. Other causes of
microcytic anemia include anemia of chronic disease, thalassemia, and sideroblastic
anemia. Anemia of chronic disease is characterized by normal or increased iron stores
in the bone marrow and a normal or elevated ferritin level; the serum iron is low, often
drastically so, and the TIBC is either normal or low. Thalassemia produces a greater
degree of microcytosis for any given level of anemia than does iron deficiency. Red
blood cell morphology on the peripheral smear is abnormal earlier in the course of
thalassemia.

1010.- Paciente con anemia microcítica leve, el aumento en la concentración


de la hemoglobina A2 sugiere el diagnóstico de:

a) Talasemia alfa
b) Rasgo trepanocitico
c) Talasemia beta
d) Esferocitosis hereditaria
e) Persistencia hereditaria de hemoglobina fetal

ESSENTIALS OF DIAGNOSIS OF THALASEMIAS:


• Microcytosis out of proportion to the degree of anemia
• Positive family history or lifelong personal history of microcytic anemia
• Abnormal red blood cell morphology with microcytes, acanthocytes,
and target cells
• In b-thalassemia, elevated levels of hemoglobin A2 or F
1011.- En un paciente alcohólico crónico se observa con frecuencia la
presencia de macrocitosis que esta relacionada con:

a) Reticulocitosis
b) Consumo de alcohol
c) Deficiencia de folato
d) Deficiencia de vitamina B 12
e) Enfermedad hepática

Folic acid is the term commonly used for pteroylmonoglutamic acid. In its reduced form
of tetrahydrofolate, it serves as an important mediator of many reactions involving
one-carbon transfers. Important reactions include the conversion of homocysteine to
methionine and of deoxyuridylate to thymidylate, an important step in DNA synthesis.
Folic acid is present in most fruits and vegetables (especially citrus fruits and green
leafy vegetables) and daily requirements of 50-100 mcg/d are usually met in the diet.
Total body stores of folate are approximately 5000 mcg, enough to supply
requirements for 2-3 months. By far the most common cause of folate deficiency is
inadequate dietary intake. Alcoholics, anorectic patients, persons who do not eat
fresh fruits and vegetables, and those who overcook their food are candidates for folate
MX-Biomedical Research Group / JD-MD Bioinformatics Labs 8
JD-MD Medicine and Technology
ENARM 11ª PARTE by JD-MD

deficiency. Reduced folate absorption is rarely seen, since absorption occurs from the
entire gastrointestinal tract. However, drugs such as phenytoin, trimethoprim-
sulfamethoxazole, or sulfasalazine may interfere with folate absorption. Folic acid
requirements are increased in pregnancy, hemolytic anemia, and exfoliative skin
disease, and in these cases the increased requirements (five to ten times normal) may
not be met by a normal diet. Patients with increased folate requirements should
receive supplementation with 1 mg/d of folic acid.

1012.- Variedad de hemopatía que presenta el cromosoma Philadelphia:

a) Leucemia promielocítica
b) Linfoma de linfocitos pequeños
c) Leucemia mieloide crónica
d) Leucemia mieloblástica M2
e) Linfoma de Burkitt

El cromosoma Filadelfia es un cromosoma anormal originado por la translocación


recíproca entre los brazos largos de los cromosomas 9 y 22, que aparece típicamente
en la leucemia mieloide crónica.

1013.- Dolor articular, exantema malar, pleuritis, anticuerpos a DNA natural


elevados y deficiencia del complemento C2:

a) Artritis reumatoide
b) Enfermedad de Raynaud
c) Vasculitis
d) Lupus eritematosos sistémico
e) Sífilis secundaria

Classification Criteria for the Diagnosis of SLE:


1. Fixed erythema, flat or raised, over the malar eminences
2. Discoid rash: Erythematous circular raised patches with adherent keratotic
scaling and follicular plugging; atrophic scarring may occur
3. Photosensitivity: Exposure to ultraviolet light causes rash
4. Oral ulcers: Includes oral and nasopharyngeal ulcers, observed by physician
5. Arthritis: Nonerosive arthritis of two or more peripheral joints, with tenderness,
swelling, or effusion
6. Serositis: Pleuritis or pericarditis documented by ECG or rub or evidence of
effusion
7. Renal disorder: Proteinuria >0.5 g/d or =3+, or cellular casts
8. Neurologic disorder: Seizures or psychosis without other causes
9. Hematologic disorder: Hemolytic anemia or leukopenia (<4000/uL) or
lymphopenia (<1500/uL) or thrombocytopenia (<100,000/uL) in the absence of
offending drugs
10. Immunologic disorder: Anti-dsDNA, anti-Sm, and/or anti-phospholipid
11. Antinuclear antibodies: An abnormal titer of ANA by immunofluorescence or an
equivalent assay at any point in time in the absence of drugs known to induce
ANAs

1014.- Lesión más frecuente provocada por fiebre reumática:

a) Insuficiencia mitral
b) Estenosis mitral
c) Insuficiencia aórtica
MX-Biomedical Research Group / JD-MD Bioinformatics Labs 9
JD-MD Medicine and Technology
ENARM 11ª PARTE by JD-MD

d) Estenosis aórtica
e) Estenosis tricúspidea
No single specific clinical manifestation or specific laboratory test unequivocally
establishes the diagnosis of rheumatic fever.
Carditis: This important finding in acute rheumatic fever is a pancarditis that involves
the pericardium, epicardium, myocardium, and endocardium. Carditis is the only
residual of acute rheumatic fever that results in chronic changes. Common
manifestations include evidence of valvular insufficiency, most frequently affecting the
mitral valve, but the mitral and the aortic valve may be affected. Isolated involvement
of the aortic valve is rare. Tricuspid valve or pulmonary valve involvement is unusual.
Valvular insufficiency is present in the acute state of the disease. Later, in
the chronic stage, scarring of the valve with either typical "fish-mouth"
abnormality or even calcified valve tissue may lead to stenosis. A combination
of insufficiency and stenosis is often found. Carditis occurs in 40-80% of patients with
rheumatic fever. In the recent outbreaks in the United States, more than 80% of
patients in one of the large series had evidence of carditis. Other manifestations of
carditis include pericarditis, pericardial effusion, and arrhythmias (usually first-degree
heart block, but third-degree or complete heart block may occur). The carditis of
rheumatic fever may be mild or very severe, leading to intractable heart failure; rarely,
surgical intervention, even in the acute stage of the disease, may be necessary if
medical management cannot control the heart failure. These patients usually have
myocardial involvement and significant valvular insufficiency.
Polyarthritis: This is the most confusing of the major criteria and probably leads to
more diagnostic errors than any of the other manifestations. The arthritis of acute
rheumatic fever is exquisitely tender. It is not uncommon for children with this form of
arthritis to refuse to allow even bed sheets or clothing to cover an affected joint. The
joints are red, warm, and swollen. The arthritis is migratory and affects several
different joints: the elbows, knees, ankles, and wrists. It rarely occurs in the fingers,
toes, or spine. It need not be symmetric. Effusions may be present. If the joint is
aspirated, a leukocytosis is usually found; polymorphonuclear leukocytes are the cells
found most frequently. However, there are no specific laboratory findings in the
synovial fluid. The arthritis does not result in chronic joint disease. After anti-
inflammatory therapy is begun, the arthritis may disappear in 12-24 hr. Untreated, it
may persist for a week or more. In many patients with early arthritis of rheumatic
fever, because of treatment with anti-inflammatory drugs, the classic migratory
polyarthritis does not develop, confusing the diagnosis.
Chorea: Sydenham chorea, a unique part of the rheumatic fever syndrome, occurs
much later than other manifestations. These choreoathetoid movements may begin
very subtly. The latent period following streptococcal pharyngitis may be as long as
several months, and the movements are often very difficult to detect at the onset.
However, careful questioning of parents and teachers usually reveals evidence of
increased clumsiness. One of the best signs of this in school-aged children is a marked
deterioration in their handwriting. Emotional lability is a frequent finding. Sydenham
chorea may affect all four extremities or may be unilateral. Although at one time it
could be seen in as many as one half of patients with acute rheumatic fever, more
recent evidence suggests that it occurs, at least in the United States, in 10% or fewer
cases. Sydenham chorea frequently is the only symptom of rheumatic fever. It usually
disappears within weeks to months. It may return, but this has become a rare
occurrence.
Erythema Marginatum: This unique rash seen in patients with rheumatic fever is
another of the major manifestations that can be very difficult to diagnose. It occurs
very infrequently, and therefore few clinicians have had extensive experience in
recognizing it. Although early in the disease it may be manifested as nonspecific pink
macules that are usually seen over the trunk, later in its fully developed form,
MX-Biomedical Research Group / JD-MD Bioinformatics Labs 10
JD-MD Medicine and Technology
ENARM 11ª PARTE by JD-MD

blanching occurs in the middle of the lesions, sometimes with fusing of the borders,
resulting in a serpiginous-looking lesion. This rash can be made worse with application
of heat, but characteristically it is evanescent. The rash does not itch. It often occurs in
patients with chronic carditis. The rash of erythema marginatum can be mistaken for
the rash seen with Lyme disease.
Subcutaneous Nodules: These lesions occur infrequently and are most commonly
observed in patients with severe carditis. These pea-sized nodules are firm and
nontender, and there is no inflammation. They are characteristically seen on the
extensor surfaces of the joints, such as the knees and elbows, and over the spine.
Minor Manifestations: The minor manifestations are much less specific but may be
necessary to confirm a diagnosis of rheumatic fever. They include the clinical findings
of fever and arthralgia. Arthralgia is present if a patient feels discomfort in a joint in the
absence of objective findings (e.g., pain, redness, warmth) on physical examination.
Fever, usually a temperature no higher than 101-102°F, may be present. High
temperature of 103-104°F requires careful re-evaluation and consideration of other
diagnoses.
The major complication of acute rheumatic fever is the development of
rheumatic valvular heart disease. None of the other manifestations results in a
chronic disease. The mitral valve is most frequently involved, but the aortic and
tricuspid valves also may be affected. The tricuspid valve usually becomes involved
only in patients who have significant mitral or aortic disease resulting in pulmonary
hypertension.

1015.- Carditis, poliartritis y corea son criterios de:

a) Thompson
b) Ranson
c) Reiter
d) Jones
e) Child

Criterios diagnósticos de Jones: Conjunto de criterios mayores (carditis,


poliartritis, corea minor, nódulos subcutáneos, eritema anular de Leiner-Lehndorff) y
menores (fiebre; artralgias; prolongación PR en el electrocardiograma; aumento de la
velocidad de sedimentación globular, proteína C reactiva o leucocitosis; signos de
infección previa por estreptococo -hemolítico; fiebre reumática previa, cardiopatía
reumática inactiva), que sirven para diagnosticar la fiebre reumática. La presencia de
dos criterios mayores o un criterio mayor y dos menores, hace altamente probable el
diagnóstico.

1016.- La fiebre reumática es una enfermedad infecciosa de tipo:

a) Degenerativa
b) Proliferativa
c) Congénita
d) Inmunológica
e) Tóxica

Despite remarkable increases in our knowledge of the biology of the group A


streptococci and of the human host and despite important observations about the
epidemiologic association between group A streptococci and the human host, the
pathogenetic mechanism responsible for the development of acute rheumatic fever
remains unknown. Two basic theories are postulated to explain the development of this
sequel to group A streptococcal pharyngitis: a toxic effect produced by an extracellular
toxin of group A streptococci on target organs such as myocardium, valves, synovium,
MX-Biomedical Research Group / JD-MD Bioinformatics Labs 11
JD-MD Medicine and Technology
ENARM 11ª PARTE by JD-MD

and brain; and an abnormal immune response by the human host. The search for the
correct hypothesis has been severely hampered by the fact that there is no adequate
animal model. The most popular hypotheses are those that postulate an abnormal
immune response by the human host to some still undefined component of group A
streptococci. The resulting antibodies might then cause the immunologic damage
leading to clinical manifestations. The latent period, usually 1-3 wk between the onset
of the actual group A streptococcal infection and the onset of symptoms of acute
rheumatic fever, lends support to an immunologic mechanism of tissue damage.
Although the specific antigen or antigens responsible for inciting such an immune
response have still to be identified, several possibilities exist. Group A Streptococcus is
a complex microorganism producing a large number of somatic and extracellular
antigens that evoke brisk immune responses. This theory is further supported by the
observation that different humans appear to respond quantitatively differently to
streptococcal antigens. For example, in vitro studies with human lymphocytes show
that individuals can be divided into high and low responders to streptococcal blastogen
A, an extracellular product of the organism. This finding is compatible with the clinical
and epidemiologic observations that not all people appear to be susceptible to
developing rheumatic fever. The possibility of an abnormal immune response is also
based on cross reactivity between group A streptococci M protein and human tissue.
The M protein is the virulence factor that is responsible for the organism's ability to
resist phagocytosis. In addition, after infections with group A streptococci, type-specific
immunity is conferred against the specific M protein type. The group A streptococcal M
protein shares certain amino acid sequences with some human tissues, and this has
been proposed as a possible source of cross reactivity between the organism and its
human host, leading to the abnormal immune response. One of the two classes of M
protein correlates with serotypes of group A streptococci that are frequently isolated
from patients with acute rheumatic fever. In patients with Sydenham chorea, common
antibodies to antigens are found in the group A streptococcal cell membrane and the
caudate nucleus of the brain. This observation further supports the concept of an
abnormal autoimmune mechanism for the central nervous system manifestations of
rheumatic fever and Sydenham chorea. An understanding of the pathogenesis of
rheumatic fever must encompass the differences in human susceptibility to the
development of acute rheumatic fever, including an unusual incidence of rheumatic
fever and rheumatic heart disease among members of certain family groups. In regard
to this genetic influence, a specific alloantigen is present on the surface of non-T
lymphocytes in 70-90% of rheumatic individuals, but fewer than 30% of "control"
nonrheumatic individuals have the marker. The marker is more common in families in
which there is an index case of rheumatic fever than in nonaffected members of
"control" families. Although humans may have genetic differences in rheumatic
susceptibility, the exact mechanism remains unknown. It is unlikely that the recent
outbreaks of acute rheumatic fever in the United States are caused by an increasingly
susceptible population based only on genetics. It is most likely that the pathogenetic
mechanism for the development of rheumatic fever after upper respiratory tract
infection with group A beta-hemolytic streptococci involves a combination of specific
characteristics of the organism and some yet incompletely defined genetic
predisposition in the human host.

1017.- Bloqueador beta no cardioselectivo:

a) Atenolol
b) Metoprolol
c) Betaxolol
d) Propanolol
e) Acebutolol

MX-Biomedical Research Group / JD-MD Bioinformatics Labs 12


JD-MD Medicine and Technology
ENARM 11ª PARTE by JD-MD

El propanolol es un beta-bloqueador no selectivo, el metoprolol es beta-bloqueador


selectivo, el labetadol es un beta-bloqueador no selectivo con selectividad alfa, el
carvedilol es un bloqueador beta y alfa.

1018.- En una mujer de 20 años con masa asintomática de 2 cms. En el polo


inferior del lóbulo tiroideo derecho, la maniobra inicial mas apropiada:

a) Biopsia excisional
b) Biopsia incisional
c) Aspiración con aguja para citología
d) Estudio de captación de yodo radioactivo
e) Examen de tiroides con ultrasonido

Nodular thyroid disease is common, occurring in about 3 to 7% of adults when


assessed by physical examination. Using more sensitive techniques, such as
ultrasound, it is present in >25% of adults. Thyroid nodules may be solitary or multiple,
and they may be functional or nonfunctional. Benign thyroid nodules are common (5 to
10% adults), particularly when assessed by sensitive techniques such as ultrasound.
The risk of malignancy is very low for macrofollicular adenomas and normofollicular
adenomas. Microfollicular, trabecular, and Hurthle cell variants raise greater concern,
and the histology is more difficult to interpret. About one-third of palpable nodules are
thyroid cysts. These may be recognized by their ultrasound appearance or based on
aspiration of large amounts of pink or straw-colored fluid (colloid). Many are mixed
cystic/solid lesions, in which case it is desirable to aspirate cellular components under
ultrasound or harvest cells after cytospin of cyst fluid. Cysts frequently recur, even
after repeated aspiration, and may require surgical excision if they are large or if the
cytology is suspicious. Sclerosis has been used with variable success but is often
painful and may be complicated by infiltration of the sclerosing agent.

1019.- Mecanismo mediante el cual la heparina previene la trombosis:

a) Inhibe la activación del factor X


b) Inhibe la activación del factor VIII
c) Inhibe la activación de los factores VIII y X
d) Incrementa la actividad de antitrombina III
e) Inhibe la agregación plaquetaria

La antitrombina, sintetizada en el hígado, inhibe los factores de la coagulación


activados: trombina, IXa, Xa, XIa, XIIa y calicreina. La antitrombina III inhibe la
trombina sólo en presencia de la heparina, ya que ésta al fijarse a la antitrombina le
induce un cambio conformacional que hace más accesible el sitio reactivo a la
trombina.

1020.- Virus conocidos como agentes etiológicos del carcinoma del cuello
uterino:

Virus Epstein-barr (EBV)


Virus herpes simple tipo 2 (HSV-2)
Virus 1 de inmunodeficiencia humana (HIV 1)
Poliovirus (PV)
Virus sarcoma Rous (RSV)

Aunque es por todos conocido que la infección por VPH tipo 16 y 18 es característica de
lesiones precancerosas para cérvix y vulva, también esta documentado que la

MX-Biomedical Research Group / JD-MD Bioinformatics Labs 13


JD-MD Medicine and Technology
ENARM 11ª PARTE by JD-MD

infección por VHS tienen un rasgo de recurrencia del 65% que se relaciona con una
mayor incidencia de cáncer de cérvix, y que además facilita el contagio por VIH.

1021.- Tratamiento de elección para síndrome disenteriforme y sospecha de


shigella Sp:

Ceftriaxona
Gentamicina
Trimetropin/Sulfametoxasol
Colimicina
Furazolidona

Siguella: su localización más frecuente es el recto-sigmoides, produce abscesos en las


criptas, diarrea con moco y sangre, con DOLOR RECTAL, prototipo de la disentería
bacilar (bacilo G -). Se adquiere a través de agua contaminada. La Siguella puede
producir síndrome urémico-hemolítico, que en el adulto es mas grave. El tratamiento
en los niños es TMP-SMX, en adultos el ciprofloxacino es la mejor opción, también se
puede dar ampicilina.

1022.- Edad idónea en la que se debe aplicar la vacuna SPR (sarampión,


parotiditis y rubéola), o triple viral:

a) 9 meses
b) 12 meses
c) 15 meses
d) Previo al ingreso a la escuela (preescolar)
e) Inició de la adolescencia

La triple viral se aplica al año y a los 6 años de edad.

1023.- En los pacientes pediátricos, la osteoporosis puede ser causada por el


uso prolongado de:

a) Isoniazida
b) Rifampicina
c) Fenobarbital
d) Trimetoprim-sulfametoxazol
e) Difenilhidantoina

Se ha informado de raquitismo y osteomalacia en quienes reciben fenilhidantoina y


fenobarbital como terapia anticonvulsiva prolongada. Con mayor frecuencia, los
fármacos inducen un estado de osteoporosis con recambio alto, a consecuencia de
disminución de la absorción intestinal de Ca+.

1024.- Se producen reacciones de temor o miedo cuando se aplican estímulos


en:

a) Hipotalamo
b) Núcleos amigdalinos
c) Hipocampo
d) Lóbulo frontal
e) Núcleo anterior

La reacción de temor puede producirse en animales conscientes, por estimulación del


hipotálamo y núcleo amigdaloide. A la inversa, las reacciones de miedo y sus
MX-Biomedical Research Group / JD-MD Bioinformatics Labs 14
JD-MD Medicine and Technology
ENARM 11ª PARTE by JD-MD

manifestaciones autónomas y endocrinas faltan cuando se ha destruido la amígdala, en


situaciones en las que serían evocadas normalmente. Hay una cantidad considerable
de pruebas de que el núcleo amigdaloide se ocupa de la codificación de las memorias
que evocan temores.

1025.- La causa más frecuente de muerte posterior a un infarto del miocardio


es la presentación de:

a) Nuevo infarto del miocardio


b) Insuficiencia cardiaca progresiva
c) Taquiarritmias ventriculares
d) Bloqueo auriculoventricular
e) Aneurisma ventricular

La causa más común de muerte en las primeras 48 hrs posteriores a un infarto agudo
del miocardio es la fibrilación ventricular (FV). Entre otras causas se mencionan la
rotura cardiaca, bombeo insuficiente debido a infarto masivo, complicaciones
mecánicas agudas como la rotura del tabique ventricular o regurgitación mitral aguda
y choque cardiogénico.

1026.- El mecanismo de daño en el lupus eritematoso generalizado más


importante es:

a) Anafilaxia
b) Citotoxicidad por anticuerpos
c) Deposito de complejos inmunes
d) Hipersensibilidad retardada
e) Daño tisular mediado por células

SLE1 is caused by interactions between susceptibility genes and environmental factors,


resulting in abnormal immune responses. The immune responses include
hyperreactivity and hypersensitivity of T and B lymphocytes and ineffective regulation
of antigen availability and of ongoing antibody responses. Hyperreactivity of T and B
cells is indicated by increased surface expression of molecules such as HLA-D and
CD40L, showing that cells are easily activated by antigens that induce first-activating
signals and by molecules that drive cells to full activation via second signals. The end
result of these abnormalities is sustained production of pathogenic autoantibodies and
formation of immune complexes that bind target tissues, resulting in (1) sequestration
and destruction of Ig-coated circulating cells; (2) fixation and cleaving of complement
proteins; and (3) release of chemotaxins, vasoactive peptides, and destructive
enzymes into tissues. Many autoantibodies in persons with SLE are directed against
DNA/protein or RNA/protein complexes such as nucleosomes, some nucleolar RNA, and
spliceosomal RNA (Table 300-1). During apoptosis these antigens migrate to cell
surfaces, where they are enclosed in blebs, and membrane phospholipids change
orientation so that antigenic portions are near the surface. Intracellular molecules
altered during cell activation or damage migrate to the cell surface. All these antigens
near or in cell surfaces probably activate the immune system to produce
autoantibodies. In individuals with SLE, phagocytosis and removal of apoptotic cells
and of immune complexes are impaired. Thus, in SLE, antigens are available; they are
presented in locations recognized by the immune system; and the antigens,
autoantibodies, and immune complexes persist for prolonged periods of time, allowing
tissue damage to accumulate to the point of clinical illness. SLE is a multigenic disease.
It is likely that alleles of multiple normal genes each contribute a small amount to the
abnormal immune responses; if enough variations accumulate, disease results. Some
predisposing genes are located in the HLA region (particularly HLA class II DR and DQ

MX-Biomedical Research Group / JD-MD Bioinformatics Labs 15


JD-MD Medicine and Technology
ENARM 11ª PARTE by JD-MD

genes, and HLA class III genes encoding C´2 and C´4). The relevant HLA DR/DQ genes
increase risk for SLE by approximately twofold if one susceptibility haplotype is present
and by four- to sixfold if two or more are present. Some proteins important in clearing
apoptotic cells play a role in genetic susceptibility; for example, homozygous
deficiencies of early components of complement Clq, C´2, and C´4 and certain alleles
of mannose-binding ligand increase the risk for SLE. Clq deficiency confers the highest
genetic risk known but is rare. There are at least five chromosomal regions
independent of HLA that contain susceptibility genes. Within one of these regions on
chromosome 1 are alleles encoding Fc? receptors that bind subsets of IgG (IgG1, -2, or
-3): African Americans inheriting one allele of Fc?RIIA have a receptor that binds the Ig
in immune complexes weakly; those persons have increased risk for lupus nephritis.
Caucasians and Asians in some populations with alleles of Fc?RIIIA that bind Ig weakly
are predisposed to SLE. A region on chromosome 16 contains genes that predispose to
SLE, rheumatoid arthritis, psoriasis, and Crohn's disease, suggesting the presence of
"autoimmunity genes" that, when interacting with other genes, predispose to different
autoimmune diseases. Thus, SLE is modified by multiple susceptibility genes, some of
which interact. There are likely to be protective gene alleles as well. These gene
combinations influence immune responses to the external and internal environment;
when such responses are too high and/or too prolonged, autoimmunity results. Female
gender is permissive for SLE; females of many mammalian species make higher
antibody responses than males. Women exposed to estrogen-containing oral
contraceptives or hormone replacement have an increased risk of developing SLE
(approximately twofold). Estradiol binds to receptors on T and B lymphocytes,
increasing activation and survival of those cells, thus favoring prolonged immune
responses. Several environmental stimuli may influence SLE1. Exposure to ultraviolet
light causes SLE flares in approximately 70% of patients, possibly by increasing
apoptosis in keratinocytes and other cells or by altering DNA and intracellular proteins
to make them antigenic. It is likely that various infections that stimulate immune
responses (antibodies and activated T lymphocytes) that cross-react with self or
responses that, as they mature, develop the ability to recognize self can promote
autoimmune responses that lead to SLE. The observation that children and adults with
SLE are more likely to be infected by Epstein-Barr Virus (EBV) than age-, gender-, and
ethnically matched controls without SLE is intriguing, because EBV activates B
lymphocytes and also contains amino acid sequences that mimic sequences on human
spliceosomes — a common autoantibody specificity in people with SLE. Thus, interplay
between genetic susceptibility, gender, and environmental stimuli may result in
autoimmunity. For maximal production of harmful autoantibodies, B cells require help
from T cells, and those functions of T and B cells are normally downregulated by
several mechanisms. In murine SLE models, many downregulating networks are
abnormal, including generation of multiple types of regulatory and natural killer T cells
and of humoral idiotypic downregulating networks. In SLE, biopsies of affected skin
show deposition of Ig at the dermal-epidermal junction (DEJ), injury to basal
keratinocytes, and inflammation dominated by T lymphocytes in the DEJ and around
blood vessels and dermal appendages. Clinically unaffected skin may also show Ig
deposition at the DEJ. In renal biopsies, the pattern of injury is important in diagnosis
and in selecting the best therapy. The World Health Organization (WHO) has classified
lupus nephritis as grade I (no histologic changes), II (proliferative changes confined to
the mesangium), III (proliferative changes in tufts of 10 to 50% of glomeruli; higher
proportions of glomeruli affected suggest worse prognosis), IV [diffuse proliferative
glomerulonephritis (DPGN) affecting >50% of glomeruli], V (predominantly
membranous changes with various degrees of proliferation), and VI (end stage, scarred
glomeruli). In addition, pathologists report the extent of inflammatory (potentially
reversible) and chronic (irreversible scarring in glomeruli, renal tubules, and blood
vessels) changes. In general, treatment for lupus nephritis is not recommended in
patients with class I or II disease or with extensive irreversible changes. In contrast,

MX-Biomedical Research Group / JD-MD Bioinformatics Labs 16


JD-MD Medicine and Technology
ENARM 11ª PARTE by JD-MD

aggressive immunosuppression is recommended for patients with class III, IV, or V


inflammatory proliferative lesions because the majority of those individuals, if
untreated, develop end-stage renal disease (ESRD) within 2 years. In children, a
diagnosis of SLE can be established on the basis of renal histology without meeting
additional diagnostic criteria. Histologic abnormalities in blood vessels are not specific
for SLE: leukocytoclastic vasculitis is most common. Lymph node biopsies show
nonspecific diffuse chronic inflammation.

1027.- Caso clínico seriado: Paciente masculino de 50 años que presenta


súbitamente dolor intenso en primer ortejo de pie derecho durante la noche
posterior, tras ingesta de alcohol. Primer enunciado con los datos clínicos
anteriores usted pensaría que el paciente cuenta con:

a) Hiperuricemia crónica sintomática


b) Síndrome de Reiter
c) Artritis reumatoide
d) Monoartritis infecciosa
e) Artritis gotosa

1028.- Segundo enunciado. Se sabe que en esta patología existe una


alteración que consiste en:

a) Disminución en la eliminación del ácido úrico


b) Complejo antígeno-anticuerpo a nivel de la articulación afectada
c) Degradación de cuerpos cetonicos
d) Reacción cruzada contra los condrocitos
e) Inflamación viral

1029.- Tercer enunciado. El tratamiento de elección es:

a) AINEs
b) Alopurinol y glucocorticoides
c) Colchicina y AINEs
d) Pirazona y alopurinol
e) Cirugía para extracción de tofos

Monosodium urate (MSU) gout is a metabolic disease most often affecting middle-aged
to elderly men and postmenopausal women. It is typically associated with an increased
uric acid pool, hyperuricemia, episodic acute and chronic arthritis, and deposition of
MSU crystals in connective tissue tophi and kidneys. Acute arthritis is the most
frequent early clinical manifestation of MSU gout. Usually, only one joint is affected
initially, but polyarticular acute gout is also seen in male hypertensive patients with
ethanol abuse as well as in postmenopausal women. The metatarsophalangeal joint of
the first toe is often involved, but tarsal joints, ankles, and knees are also commonly
affected. In elderly patients, finger joints may be inflamed. Inflamed Heberden's or
Bouchard's nodes may be a first manifestation of gouty arthritis. The first episode of
acute gouty arthritis frequently begins at night with dramatic joint pain and swelling.
Joints rapidly become warm, red, and tender, and the clinical appearance often mimics
a cellulitis. Early attacks tend to subside spontaneously within 3 to 10 days, and most
of the patients do not have residual symptoms until the next episode. Several events
may precipitate acute gouty arthritis: dietary excess, trauma, surgery, excessive
ethanol ingestion, adrenocorticotropic hormone (ACTH) and glucocorticoid withdrawal,
hypouricemic therapy, and serious medical illnesses such as myocardial infarction and
stroke. After many acute mono- or oligoarticular attacks, a proportion of gouty patients
may present with a chronic nonsymmetric synovitis, causing potential confusion with

MX-Biomedical Research Group / JD-MD Bioinformatics Labs 17


JD-MD Medicine and Technology
ENARM 11ª PARTE by JD-MD

rheumatoid arthritis. Less commonly, chronic gouty arthritis will be the only
manifestation and, more rarely, the disease will manifest as inflamed or noninflamed
periarticular tophaceous deposits in the absence of chronic synovitis. Women represent
only 5 to 17% of all patients with gout. Premenopausal gout is a rare occurrence and
accounts for only about 17% of all women with gout; it is seen mostly in individuals
with a strong family history of gout. A few kindreds of precocious gout in young
females caused by decreased renal urate clearance and renal insufficiency have been
described. Most women with gouty arthritis are postmenopausal and elderly, have
arterial hypertension causing mild renal insufficiency, and are usually receiving
diuretics. Also, most of these patients have underlying degenerative joint disease, and
inflamed tophaceous deposits may be seen on Heberden's and Bouchard's nodes. The
mainstay of treatment during an acute attack is the administration of an anti-
inflammatory drug such as colchicine, nonsteroidal anti-inflammatory drugs (NSAIDs),
or glucocorticoids depending on the age of the patient and comorbid conditions. Both
colchicine and NSAIDs may be quite toxic in the elderly, particularly in the presence of
renal insufficiency and gastrointestinal disorders. In elderly patients, one may favor the
use of intraarticular glucocorticoid injections for attacks involving one or two larger
joints or ice pack applications along with lower oral doses of colchicine for gouty
synovitis affecting small joints. Colchicine given orally is a traditional and effective
treatment, if used early in the attack, in at least 85% of patients. One tablet (0.6 mg) is
given every hour until relief of symptoms or gastrointestinal toxicity occurs, or a total
of four to eight tablets may be given in accordance with the age of the patient. The
drug must be stopped promptly at the first sign of loose stools, and symptomatic
treatment must be given for the diarrhea. Intravenous colchicine is sometimes used
and can reduce, though not eliminate, the gastrointestinal side effects. Intravenous
colchicine is most reliable for pre- or postoperative prophylaxis in 1- to 2-mg doses
when patients cannot take medications orally. Life-threatening colchicine toxicity and
sudden death have been described with the administration of >4 mg/d intravenously.
The intravenous dose for acute gouty arthritis is 1 to 2 mg given slowly through an
established venous line over 10 min in a soluset, and two additional doses of 1 mg
each may be given at 6-h intervals, but the total dose should never exceed 4 mg.
NSAIDs are affective in ~90% of patients, and the resolution of signs and symptoms
usually occurs in 5 to 7 days. The most effective drugs are those with a short half-life
and include indomethacin, 25 to 50 mg tid, ibuprofen, 800 mg tid, or diclofenac, 50 mg
tid. Cyclooxigenase-2 highly selective inhibitors are probably equally effective but with
less short-term gastrointestinal toxicity. Oral glucocorticoids such as prednisone, 30 to
50 mg/d as the initial dose and tapered over 5 to 7 days, a single intravenous dose of
methylprednisolone, 7 mg of betametasone, or 20 to 40 mg of intraarticular
triamcinolone acetonide have been equally effective. ACTH2 as an intramuscular
injection of 40 to 80 IU in a single dose or every 12 h for 1 to 2 days is effective in
patients with acute polyarticular refractory gout or with a contraindication for using
colchicine or NSAIDs. Attempts to normalize serum uric acid to <300 umol/L (5.0
mg/dL) to prevent recurrent gouty attacks and eliminate tophaceous deposits entail a
commitment to long-term hypouricemic regimens and medications that generally are
required for life. Hypouricemic therapy should be considered when the hyperuricemia
cannot be corrected by simple means (control of body weight, low-purine diet, increase
in liquid ingestion, limitation of ethanol intake, and avoidance of diuretic use). The
decision to initiate hypouricemic therapy is usually made taking into consideration the
number of acute attacks, family history of gout, presence of MSU1 tophaceous
deposits, uric acid excretion >800 mg per 24 hours, presence of uric acid stones, and
risk for acute uric acid nephropathy during chemotherapy for myeloproliferative
disorders. Uricosuric agents, such as probenecid, can be used in patients with good
renal function who underexcrete uric acid, with <600 mg in a 24-hour urine sample.
Urine volume must be maintained by ingestion of 1500 mL of water every day.
Probenecid can be started at a dosage of 200 mg twice daily and increased gradually

MX-Biomedical Research Group / JD-MD Bioinformatics Labs 18


JD-MD Medicine and Technology
ENARM 11ª PARTE by JD-MD

as needed up to 2 g in order to maintain a serum uric acid level <300 umol/L (5


mg/dL). Probenecid is the drug of choice to treat elderly patients with hypertension and
thiazide dependence; however, probenecid is not effective with a renal creatinine
clearance <1 mL/s. These patients may require allopurinol or benzbromarone (not
available in the United States), which is another uricosuric drug that is effective in
patients with renal failure and who are receiving diuretics. Allopurinol is the best drug
to lower serum urate in overproducers, stone formers, and patients with advanced
renal failure. It can be given in a single morning dose, 300 mg initially and increasing
up to 800 mg if needed. In most patients, it is not necessary to start at a lower dose;
however, in patients with renal failure, the dosage should be adjusted depending on
the serum creatinine concentration in order to minimize side effects. Patients with
frequent acute attacks may require lower initial doses to prevent exacerbations.
Toxicity of allopurinol has been recognized increasingly in patients with renal failure
who use thiazide diuretics and in those patients allergic to penicillin and ampicillin. The
most serious side effects include skin rash with progression to life-threatening toxic
epidermal necrolysis, systemic vasculitis, bone marrow suppression, granulomatous
hepatitis, and renal failure. Urate-lowering drugs should not be initiated during acute
attacks. This is especially important in patients who have refractory acute arthritis or
who had a flare-up previously with hypouricemic drugs. Colchicine prophylaxis in doses
of 0.6 mg one to two times daily is usually continued, along with hypouricemic therapy,
until the patient is normouricemic and without gouty attacks for 3 months. However,
prophylactic colchicine treatment may be necessary as long as tophi are present.
Recombinant urate oxidase uricase can be used in the short-term prophylaxis and
treatment of chemotherapy-associated hyperuricemia in patients with
lymphoproliferative and myeloproliferative disorders.

1030.- Paciente masculino de 37 años de edad que acude a urgencias por


fiebre, dolor abdominal difuso. A la exploración física se encuentra Giordano
bilateral. EGO: leucocitos 10 x campo, bacterias ++, Rx de abdomen con
silueta renal, presencia de aire de forma bilateral. Con estos datos establece
el Dx de:

a) Litiasis urinaria
b) Infección de vías urinarias
c) Pielonefritis enfisematosa
d) Cistitis
e) Prostatitis

Formación de gas, en el parenquimatoso renal (rayos X de abdomen o TAC), en el


contexto de una pielonefritis aguda, típica en pacientes diabéticos (fermentación mixta
de la glucosa). Su mortalidad es del 100% si no se somete el paciente a un tratamiento
y del 9 al 40% con un tratamiento óptimo. Este consiste en un drenaje inmediato y en
la administración de antibióticos sensibles a los gérmenes gram-negativos, que son los
habitualmente responsables.

1031.- Paciente femenino de 27 años con antecedente de infección de vías


urinarias recurrentes, por cultivo se identifico Proteus. Hace 24 horas cursa
con un lito en vía urinaria. El componente más probable de este lito es:

a) Ácido úrico
b) Pirofosfato de calcio
c) Hidroxiapatita
d) Estruvita
e) Fosfato de calcio

MX-Biomedical Research Group / JD-MD Bioinformatics Labs 19


JD-MD Medicine and Technology
ENARM 11ª PARTE by JD-MD

E coli is the most common cause of acute uncomplicated community-acquired UTI,


accounting for ~ 95% of cases. In recurrent infections, especially in association with
structural abnormalities, the relative frequency increases for infection caused by other
microorganisms, such as Proteus species, Klebsiella species, Enterobacter species, and
enterococci. Nosocomial acquisition, especially in association with urinary
catheterization, is often caused by E coli, Pseudomonas species, or other gram-
negative nosocomial microorganisms (see section on "Nosocomially Acquired Infection"
later in this chapter). Urea-splitting microorganisms, such as Proteus species,
are associated with calculus formation. S saprophyticus accounts for 5-15% of
acute cystitis in young sexually active females. Type 2 adenovirus may cause acute
hemorrhagic cystitis in children and is more common in young boys than in girls.

1032.- Paciente femenino de 32 años con rigidez matutina articular en ambas


manos de 45 minutos de duración, así como poliartritis simétrica de 8
semanas de evolución. En laboratorios se detecta VSG aumentada.
Radiograficamente se aprecian lesiones óseas en marcos articulares en
ambas manos. Por los datos clínicos y paraclínicos usted puede concluir que
la paciente padece:

a) Poliartritis infecciosa
b) Fiebre reumática
c) Osteoartritis
d) Artritis reumática
e) Artropatía por deposito de cristales

Rheumatoid arthritis (RA) is probably caused by a pathologic immune response in a


genetically predisposed person to an environmental insult, typically a viral or bacterial
infection. Epidemiologic studies show that genes encoding the class II major
histocompatibility antigens are linked to clinical features of RA. The HLADR4 and DR1
proteins present foreign and self-antigens to T cells. These molecules are presumed to
play a direct role in the etiology of this autoimmune disease by presenting an
“arthritogenic” viral or bacterial antigen to T cells. However, no organism has been
definitively linked to the etiology of RA. The use of antibiotics is controversial, and
antibiotics may have direct anti-inflammatory effects rather than acting through
antibacterial activity. Other genes of the immune, endocrine, and neural systems may
contribute to the pathogenesis of RA. The precise pathophysiologic cascade is not yet
defined. RA is an autoimmune inflammatory disease in which immunosuppressive
drugs constitute the mainstay of therapy. Certain cytokines, such as tumor necrosis
factor (TNF) and interleukin-1 (IL-1), appear to play important roles, as inhibitors of
these molecules decrease disease activity. Nonsteroidal anti-inflammatory drugs
(NSAIDs) act to inhibit the enzymes that produce inflammatory prostaglandins,
particularly thromboxanes and leukotrienes. The newer NSAIDs celecoxib (Celebrex)
and rofecoxib (Vioxx) preferentially inhibit the cyclo-oxygenase-2 (COX-2) enzyme that
produces these inflammatory molecules. Omega-3 fatty acids and certain botanicals
such as ginger and turmeric also may act through decreasing the production or activity
of inflammatory prostaglandins. The neural, endocrine, and immune systems all share
communication molecules that interact extensively. Molecules from the hypothalamic-
pituitary-adrenal axis, particularly cortisol and corticotropin-releasing factor (CRF), and
from the sympathetic-adrenal-medullary system are linked to disease activity in RA.
Corticosteroid drugs have powerful disease-suppressing activity, with equally powerful
side effects such as osteoporosis. Prolactin and the estrogenic and androgenic sex
hormones have been postulated to play roles as well. Other environmental factors such
as nutrition, coffee, and tobacco also may contribute to the increased risk of RA. Stress
and psychological factors have been linked to the etiology of RA and to disease
exacerbations. In one study, psychological factors and depression accounted for at

MX-Biomedical Research Group / JD-MD Bioinformatics Labs 20


JD-MD Medicine and Technology
ENARM 11ª PARTE by JD-MD

least 20% of disability in patients with RA, greater than the 14% attributable to
articular signs and symptoms. In another study, helplessness had a direct effect on
disease activity.

1033.- Caso clínico seriado: Un varón de 28 años en quien se implantó una


prótesis valvular mitral, que presenta hemiparesia corporal de aparición
súbita. Primer enunciado: El diagnóstico más probable es:

a) Crisis de Stoke-Adams
b) Embolia cerebral
c) Hemorragia cerebral intraparenquimatosa
d) Trombosis cerebral
e) Hemorragia subaracnoidea

1034.- Segundo enunciado: La causa más probable del cuadro clínico actual
del paciente es:

a) Bloqueo auriculoventricular de segundo grado


b) Extrasístoles ventriculares
c) Dosis insuficiente de anticoagulante
d) Bloqueo auriculoventricular de tercer grado
e) Sobredosificación de anticoagulante

1035.- Tercer enunciado: El tratamiento inmediato de este paciente se debe


llevar a cabo con:

a) Warfarina
b) Plasma fresco congelado
c) Heparina
d) Marcapaso temporal
e) Atropina

Intracardiac thrombi, which complicate many types of heart disease, are important
because of their potential to embolize. Those in the left atrium and ventricle may
embolize to the systemic circulation and are a significant cause of stroke. Acute
anticoagulation can be a temporary bridge to chronic anticoagulation in selected
clinical settings. Acute anticoagulation can be achieved with either unfractionated
heparin or low-molecular-weight heparin. The purpose of replacing diseased native
heart valves is primarily to improve hemodynamics and hence functional status and
long-term survival. The first prosthetic valves were mechanical, and although the
hemodynamic status improved following implantation, thromboembolic complications
occurred and caused such disastrous complications as stroke. Initially it was unclear
whether thromboembolism was caused by the introduction of a foreign body into the
circulation or by the underlying pathology. Thromboembolic risk continues well after
endothelialization of the sewing ring has occurred, suggesting that the mechanical
prosthesis or underlying cardiac abnormality is the cause. Whatever the mechanism, it
is apparent that these complications can be reduced with anticoagulation. Because a
mechanical prosthesis necessitates life-long anticoagulation, it is important to evaluate
patients for hemorrhagic risk prior to valve surgery. For women of child-bearing age, an
important consideration is the type of valve (mechanical or biologic) to be implanted
because of the risks of anticoagulation with pregnancy. When using anticoagulation to
reduce the thromboembolic complications associated with prosthetic valves, however,
it is important to recognize that no antithrombotic regimen guarantees a
thromboembolus-free course or a hemorrhage-free valve implant.

MX-Biomedical Research Group / JD-MD Bioinformatics Labs 21


JD-MD Medicine and Technology
ENARM 11ª PARTE by JD-MD

1036.- Caso clínico seriado: Lactante de 10 meses tiene un padecimiento de


48 horas de evolución manifestado por fiebre, vómitos, somnolencia,
hiporexia y tos. Súbitamente presenta crisis convulsivas tónico-clónicas. A la
exploración física hay rigidez de nuca, signo de Brudzinsky positivo,
espasticidad muscular e hiperreflexia; en el tórax hay estertores crepitantes
bilaterales. La biometría hemática muestra 25,000 leucocitos con predominio
de segmentados, 58,000 plaquetas; sodio 140 mEq/L; calcio 10 mg/dl;
magnesio 2 mg/dl y glucosa 90 mg/dl. Primer enunciado: Antes de confirmar
el diagnostico, el tratamiento del paciente debe iniciarse con:

a) Carbamacepina y penicilina
b) Ceftriaxona y fenobarbital
c) Gluconato de calcio y sulfato de magnesio
d) Ceftriaxona y sulfato de magnesio
e) Sulfato de magnesio y fenobarbital

1037.- Segundo enunciado: La etiología más probable de la enfermedad de


este paciente es:

a) Parasitaria
b) Febril
c) Micotica
d) Bacteriana
e) Viral

1038.- Tercer enunciado: El edema cerebral que presenta el paciente se


deberá manejar con:

a) Fenobarbital
b) Furosemida
c) Dexametasona
d) Fenitoina
e) Solución glucosada hipertonica

In otherwise healthy children, the three most common organisms causing


hematogenously acquired acute bacterial meningitis are Streptococcus pneumoniae,
Neisseria meningitidis, and Haemophilus influenzae type b (Hib). Although Hib was by
far the most common bacterial cause before availability of the Hib conjugate vaccine,
the current likelihood of Hib meningitis in a child who has received at least two doses of
vaccine is extraordinarily lower. In surveillance studies conducted by the Centers for
Disease Control and Prevention (CDC), the incidence of Hib disease declined by 95%
from 1987 to 1993 in children younger than 5 years (41 cases per 100,000 in 1987 to 2
per 100,000 in 1993). Meningitis due to Hib has a peak incidence in the fall and winter
and occurs more commonly in blacks. Pneumococcal meningitis and meningococcal
meningitis also occur more commonly during the winter; meningococcal disease can be
endemic or epidemic. The annual incidence of bacterial meningitis in children younger
than 5 years, as assessed by the CDC, has been relatively stable for both N.
meningitidis (about 4 to 5 cases per 100,000) and S. pneumoniae (about 2.5 per
100,000) since 1980. Seven of the >90 pneumococcal serotypes (4, 6B, 9, 14, 18F,
19F, and 23F) account for >80% of invasive disease in children from developed
countries; serotypes 5 and 1 are also prevalent in developing countries.[3] Serogroups
B and C are the most common serogroups of N. meningitidis causing invasive infections
in the American continent, although group Y strains now account for 20% to 25% of
cases. Children with a basilar skull or cribriform fracture and a cerebrospinal fluid (CSF)
leak have greater risk for pneumococcal meningitis, as do children with asplenia

MX-Biomedical Research Group / JD-MD Bioinformatics Labs 22


JD-MD Medicine and Technology
ENARM 11ª PARTE by JD-MD

(anatomic or functional) or human immunodeficiency virus (HIV) infection. Deficiencies


in terminal components of complement lead to greater risk for meningococcal
infection. Common causes of meningitis after penetrating head trauma or neurosurgery
are staphylococcal species, streptococci, and gram-negative enteric bacilli, especially
Escherichia coli, Klebsiella spp., and Pseudomonas aeruginosa. These organisms are
also associated with meningitis related to a dermal sinus or embryopathy of the
neurenteric canal. Nontypable H. influenzae meningitis is associated with
immunoglobulin deficiencies, and Listeria meningitis with cellular immune defects;
Listeria meningitis occurs occasionally in immunocompetent infants and children as
well. Salmonella spp. rarely cause meningitis in immunocompetent children beyond the
neonatal period, and some cases have been linked to reptile pets. The presence of a
ventriculoperitoneal shunt is a risk factor for meningitis; ventriculitis related to shunt
contamination and skin organisms are usually causative. Isolation of an organism other
than pneumococcus, meningococcus, or Hib from the CSF of a child older than 2
months always requires an explanation or evaluation for unusual host susceptibility.
Children with recurrent pneumococcal or meningococcal infections also should undergo
thorough investigation, including neuroimaging studies. Symptoms and signs of
bacterial meningitis depend somewhat on the age of the patient and duration of the
illness. Often, clinical manifestations are nonspecific and are not readily distinguished
from those of self-limited viral infection. Inconsolable crying is a nonspecific symptom
of serious infection in infants. In many patients, the initial symptoms consist of fever,
irritability, nausea, vomiting, and diarrhea. An infant may become progressively more
irritable and lethargic, may refuse feedings, and may manifest increasingly less
interaction with caregivers. Grunting respirations indicate a critically ill infant.
Older children may complain of headache, vomiting, back pain, myalgia, and
photophobia; may be confused or disoriented; and may verbalize specifically that the
neck is stiff or sore. Seizures are noted in up to 20% to 30% of patients before hospital
admission or early in the course of the illness and tend to occur more frequently in
pneumococcal or Hib meningitis than in meningococcal disease. The level of
consciousness at the time of treatment is an important prognostic sign; outcome for
children who are irritable or lethargic is superior to that for children who are comatose.
[9]
On physical examination, the fontanelle of an infant may be bulging, presumably
indicating increased intracranial pressure; this sign is neither highly sensitive nor
specific for meningitis but always requires evaluation. The infant demonstrates
diminished activity and may show little interest in the environment during the
examination. More specific physical findings of meningeal inflammation are Kernig and
Brudzinski signs and neck stiffness. These occur more commonly in children older than
12 to 18 months. A sixth cranial nerve palsy suggests increased intracranial pressure;
seventh and third cranial nerve dysfunctions have also been observed. Papilledema is
uncommon in a child with uncomplicated meningitis and, if present, suggests another
diagnosis (brain abscess, epidural or subdural empyema, another cause of increased
intracranial pressure) or a complication of meningitis (venous sinus thrombosis).
Unilateral weakness (hemiparesis) is a result of an infarction, usually associated with
vasculitis or spasm of a cerebral artery. In an occasional older child, ataxia is the major
complaint. A petechial or purpuric rash and shock are classically associated with
meningococcal meningitis but also can occasionally be caused by H. influenzae or S.
pneumoniae. A maculopapular rash, which is difficult to distinguish from a more
common enteroviral exanthem, can occur in up to 15% of children with
meningococcemia. Focal infections such as pneumonia, pyogenic arthritis, buccal
cellulitis, pericarditis, and endophthalmitis can be present concurrently with bacterial
meningitis; their presence should not discourage evaluation of CSF unless the severity
of the condition warrants deferral of the test. Children with bacterial meningitis present
with the following three basic patterns of illness: (1) the most common, an insidious
form with nonspecific symptoms that progress over 2 to 5 days before meningitis is

MX-Biomedical Research Group / JD-MD Bioinformatics Labs 23


JD-MD Medicine and Technology
ENARM 11ª PARTE by JD-MD

diagnosed; (2) a more rapid form, in which symptoms and signs of meningitis progress
over the course of 1 or 2 days; and (3) a fulminant form, with rapid deterioration and
shock early in the course of illness. Host and bacterial factors influence the type of
presentation. Recognizing the infant or child with meningitis or other invasive bacterial
infection among the large numbers of children with febrile illnesses due to viral or
uncomplicated bacterial infections of the upper respiratory tract requires expertise as
well as careful observation and examination. The differential diagnosis in children with
fever and alteration in CNS function includes other infections, such as viral meningitis,
encephalitis, brain abscess, and subdural or epidural abscess; Lyme disease; rickettsial
or Mycoplasma infections, and leptospirosis. Initial empirical therapy using cefotaxime
or ceftriaxone plus vancomycin (60 mg/kg/day divided q6h) is prudent for children and
infants 1 month and older. Limited clinical data preclude recommendations regarding
rifampin combined with a cephalosporin. In the Hib vaccine era, ampicillin plus
chloramphenicol is not an optimal combination for initial therapy because of its
probable inadequacy against resistant pneumococcal strains. Once an organism has
been identified and the antimicrobial susceptibility pattern is known, antibiotic therapy
can be simplified or modified. Penicillin G can be used to complete therapy for
pneumococcal or meningococcal meningitis due to susceptible organisms. Cefotaxime
or ceftriaxone is continued for penicillin-resistant pneumococci that are susceptible to
these two agents (MIC =0.5 µg/mL). Although many patients with isolates requiring an
MIC of 1.0 µg/mL for third-generation cephalosporins have been treated successfully,
this fact should not be relied on. It is prudent to repeat the CSF examination after 24 to
48 hours of therapy in patients with pneumococcal infection to document a sterile
culture and negative Gram stain. If results of the second CSF culture or Gram stain is
positive, or a pneumococcal isolate requires an MIC of 1.0 µg/mL or greater for
extended-spectrum cephalosporins, vancomycin (if not previously begun) with or
without rifampin should be added.[33] Rifampin should be added if the patient is
already receiving vancomycin. Vancomycin penetration into CSF during meningitis may
be unpredictable, and thus, peak serum levels should be in the upper therapeutic
range (35 to 40 µg/mL). Chloramphenicol is an option for treatment of bacterial
meningitis caused by ampicillin-resistant Hib or in the child with a history of
anaphylaxis or respiratory distress associated with penicillin or other ß-lactam
antibiotics. Because the pharmacokinetics of chloramphenicol is variable, serum
concentrations should be monitored to ensure that safe and effective levels are
achieved.[34] Ideal peak values are 15 to 30 µg/mL at 60 to 120 minutes after the
infusion is complete. Levels exceeding 30 µg/mL are associated with bone marrow
suppression; levels greater than 50 to 80 µg/mL have been related to ”gray baby“
syndrome (which is not confined to infants) and impaired myocardial contractility.
Chloramphenicol is best avoided in children with septic shock. Additionally, several
drugs affect the metabolism of chloramphenicol: Concurrent administration of
phenobarbital and rifampin lowers the chloramphenicol metabolism, whereas
administration of phenytoin raises it. Other potentially useful antimicrobial agents for
the treatment of bacterial meningitis are cefepime and meropenem, both of which
have been shown to be equivalent to third-generation cephalosporins in treatment of
children infected with common meningeal pathogens. The efficacy of these antibiotics
against ß-lactam–resistant pneumococci has not been defined, although it appears not
to be superior to that of cefotaxime or ceftriaxone. The use of imipenem-cilastatin is
not recommended in children with CNS infection because of potential epileptogenic
activity of imipenem in the pediatric population. Because of the increasing rate of
isolation of multiple-drug-resistant pneumococci worldwide, there is an urgent need to
develop antibiotics with different mechanisms of action against these strains.
Accordingly, new fluoroquinolones are currently undergoing clinical evaluation in
children with bacterial meningitis. Modulating the host response to infection may be
beneficial in decreasing some sequelae of meningitis. Prospective studies have shown
that dexamethasone therapy initiated just before or concurrently with the first dose of

MX-Biomedical Research Group / JD-MD Bioinformatics Labs 24


JD-MD Medicine and Technology
ENARM 11ª PARTE by JD-MD

intravenous antibiotics significantly diminishes the incidence of neurologic and


audiologic deficits due to Hib meningitis.

1039.- El síndrome de Horner presenta:

a) Midriasis, catarata y aumento de la presión intraocular


b) Dolor ocular, ptosis y midriasis
c) Miosis, aniridia y atrofia óptica
d) Ptosis, miosis y enoftalmos
e) Enoftalmos, catarata y ptosis

Síndrome producido por una lesión del simpático cervical en cualquier punto de su
recorrido; se manifiesta como miosis, ptosis y enoftalmos en el ojo del lado afecto.

1040.- Antimicótico perteneciente al grupo de los macrolidos que es de


aplicación parenteral:

a) Anfotericina B
b) Vancomicina
c) Fluconazol
d) Miconazol
e) Ketoconazol

La anfotericina B y la nistatina son antimicóticos que alteran la permeabilidad de la


membrana celular y son antibioticos macrolidos polienicos. El ketoconazol, miconazol y
el clortrimazol son imidazoles. EL fluconazol e itraconazol son triazoles.

1041.- Edad más frecuente en la que se presenta el eritema nodoso:

a) 5 a 15 años
b) 20 a 40 años
c) 40 a 50 años
MX-Biomedical Research Group / JD-MD Bioinformatics Labs 25
JD-MD Medicine and Technology
ENARM 11ª PARTE by JD-MD

d) Mayores de 50 años
e) Lactantes menores

Erythema nodosum is an uncommon reaction pattern seen mainly on the anterior tibial
areas of the legs. It appears as erythematous nodules in successive crops and is
preceded by fever, malaise, and arthralgia. Bilateral red, tender, rather well-
circumscribed nodules are seen mainly on the pretibial surface of the legs but also on
the arms and the body. Later, the flat lesions may become raised, confluent, and
purpuric. Only a few lesions develop at one time. Careful clinical and laboratory
examination is necessary to determine the cause of this toxic reaction pattern. The
following tests should be performed: complete blood cell count, erythrocyte
sedimentation rate, urinalysis, serologic test for syphilis, chest roentgenogram, and
specific skin tests, as indicated. The causes of erythema nodosum are streptococcal
infection (rheumatic fever, pharyngitis, scarlet fever, arthritis), fungal infection
(coccidioidomycosis, trichophyton infection), pregnancy, sarcoidosis, lymphogranuloma
venereum, syphilis, chancroid, drugs (contraceptive pills, sulfonamides, iodides,
bromides), and, rarely, tuberculosis. The disorder occurs predominantly in
adolescent girls and young women.

1042.- La complicación neurológica más frecuente en el recién nacido con


sepsis es:

a) Neumonía intraventricular
b) Meningitis
c) Infarto cerebral
d) Hidrocefalia
e) Absceso cerebral

The etiologic agents responsible for septicemia vary according to age and immunologic
status of the patient and the place of microbial acquisition (i.e., community or hospital
setting). In the 1980s and early 1990s, before the widespread implementation of
intrapartum chemoprophylaxis, group B streptococcus (GBS) was the predominant
etiologic agent of neonatal septicemia in the United States. Escherichia coli,
enterococcus, Listeria monocytogenes, non–group D a-hemolytic streptococcus, and
nontypable Haemophilus influenzae are other microorganisms responsible for sepsis
neonatorum. Early-onset neonatal septicemia can have a fulminant course and high
case-fatality rate. GBS and Listeria strains can also cause late-onset septicemia and
meningitis. Nosocomially acquired infection, which appears between the fourth
postnatal day and the time of hospital discharge, can be caused by a variety of
organisms, including coagulase-negative staphylococci, Staphylococcus aureus, gram-
MX-Biomedical Research Group / JD-MD Bioinformatics Labs 26
JD-MD Medicine and Technology
ENARM 11ª PARTE by JD-MD

negative enteric or environmental bacilli, and Candida spp. Herpes simplex virus,
enteroviruses (coxsackievirus and echovirus), and adenovirus must be considered in
the differential diagnosis of a “septic” neonate if (1) no improvement is observed after
initiation of antibiotic therapy and (2) bacterial cultures are sterile. Infants in the
community are exposed to pathogens as maternal protection wanes. Thus, they can
have septicemia due to H. influenzae type b (Hib; in areas where immunization with the
conjugate vaccine is not routine), Streptococcus pneumoniae, and Neisseria
meningitidis. Septicemia in infants younger than 3 months can be caused by either
neonatal or non-neonatal pathogens. Additionally, S. aureus and Salmonella spp. cause
invasive disease. Occasionally, viruses such as respiratory syncytial virus,
enteroviruses, and adenovirus can cause clinical findings indistinguishable from those
in bacterial infection. Sepsis not associated with an obvious site of infection is less
common in patients without underlying disorders who are older than 5 years of age.
Meningococcemia can occur in this age group, and acute disseminated group A
streptococcal and staphylococcal infection can occur in otherwise healthy children.
Patients with S. aureus septicemia usually have primary or secondary foci of infection
in the lung, bone, joint, heart, or brain. Children with invasive streptococcal infection
may or may not have primary soft tissue infection or pharyngitis; secondary sites of
infection are common. Rocky Mountain spotted fever is another important diagnosis to
consider in a child with sepsis syndrome and a rash that is maculopapular, petechial, or
both. Ehrlichiosis also can cause SIRS with or without rash. In infants and children with
nosocomial infection and in patients with primary or secondary immunodeficiency, a
broad array of common and uncommon bacteria cause sepsis. Certain conditions can
predispose children to septicemia caused by specific pathogens: Asplenia or sickle cell
disease can predispose to septicemia due to S. pneumoniae or Salmonella septicemia;
nephrotic syndrome to S. pneumoniae septicemia or peritonitis; and human
immunodeficiency virus (HIV) infection to S. pneumoniae, Hib, S. aureus, or
Pseudomonas aeruginosa septicemia. Viridans streptococci have been increasingly
recognized as common etiologic agents of severe sepsis syndrome in neutropenic
patients, particularly those treated with high doses of cytarabine. Cytomegalovirus
infection must be considered in the immunocompromised host with a clinical sepsis
syndrome. Septicemia caused by most organisms has a typical clinical course,
progressing from fever and acute inflammation to shock and multiorgan dysfunction
syndrome (MODS), with attendant physiologic derangements. Progression can be
insidious (e.g., coagulase-negative staphylococcal infection or typhoid fever) or
fulminant (e.g., meningococcemia, streptococcal or staphylococcal toxic shock).

Early-onset bacterial infections appear most commonly on day 1 of life, and the
majority of cases appear at less than 12 hours. Respiratory distress due to pneumonia
is the most common presenting sign. Other features include unexplained low Apgar
MX-Biomedical Research Group / JD-MD Bioinformatics Labs 27
JD-MD Medicine and Technology
ENARM 11ª PARTE by JD-MD

scores without fetal distress, poor perfusion, and hypotension. Late-onset bacterial
infection (at more than 5 days of age) presents in a more subtle manner, with poor
feeding, lethargy, hypotonia, temperature instability, altered perfusion, new or
increased oxygen requirement, and apnea. Late-onset bacterial sepsis is more
often associated with meningitis or other localized infections. Low total white
counts, absolute neutropenia (< 1000/mL), and elevated ratios of immature to mature
neutrophils are suggestive of neonatal bacterial infection. Thrombocytopenia is also a
common feature. Other laboratory signs are hypoglycemia or hyperglycemia with no
change in glucose administration, unexplained metabolic acidosis, and elevated C
reactive protein. In early-onset bacterial infection, pneumonia is invariably present;
chest x-ray films show infiltrates, but these infiltrates cannot be distinguished from
those resulting from other causes of neonatal lung disease. Definitive diagnosis is
made by positive cultures from blood, CSF, and the like. Early-onset infection is most
often caused by group B b-hemolytic streptococci (GBS) and gram-negative enteric
pathogens (most commonly E coli). Another organism to consider is Haemophilus
influenzae. Late-onset sepsis is caused by coagulase-negative staphylococci (most
common in infants with indwelling central venous lines), Staphylococcus aureus, GBS,
Enterococcus, Pseudomonas, and other gram-negative organisms.

1043.- Caso clínico seriado: Un lactante de 10 meses es llevado a consulta


por una tía materna por presentar muy bajo peso y deformidad de las
extremidades inferiores, que se encuentran cóncavas hacia adentro. El
paciente permanece durante todo el día dentro de su casa. Al realizar la
exploración física se aprecia que existe craneotabes, retraso en la dentición y
se corrobora la deformidad de las extremidades inferiores. Primer enunciado:
el diagnostico mas probable es:

a) Avitaminosis B
b) Escorbuto
c) Avitaminosis A
d) Raquitismo
e) Pelagra

1044.- Segundo enunciado: Para apoyar el diagnostico, en las radiografías de


los huesos largos de este lactante se debe observar:

a) Imagen de vidrio despulido en las diáfisis


b) Hiperdensidad subperiostica que rodea como un huso a la diáfisis
c) Epífisis engrosadas y forma de cúpula en la superficie articular
d) Lesiones líticas en toda la porción diafisiaria
e) Imagen de deshilachamiento en la diafisis

1045.- Tercer enunciado: El tratamiento adecuado para este lactante consiste


en suministrarle:

a) Retinoides
b) Calciferol
c) Ácido Ascórbico
d) Vitaminas del complejo B
e) Niacina

Vitamin D requirements are normally met primarily from ultraviolet radiation of


dehydrocholesterol in the skin with the formation of cholecalciferol (vitamin D3 ).
Similarly, vitamin D2, or ergocalciferol, is derived from radiation of ergosterol. Vitamin
D is transported from the skin to the liver attached to a specific carrier protein. The

MX-Biomedical Research Group / JD-MD Bioinformatics Labs 28


JD-MD Medicine and Technology
ENARM 11ª PARTE by JD-MD

primary dietary source is vitamin D–fortified milk and formulas. Egg yolk and fatty fish
contain some vitamin D. Vitamin D absorption depends on normal fat absorption.
Absorbed vitamin D is transported to the liver in chylomicrons. Vitamins D2 and D3
undergo 25-hydroxylation in the liver and then 1a-hydroxylation in kidney proximal
tubules to yield 25-hydroxycholecalciferol (calcifediol) and calcitriol, respectively. PTH
activates the 1a-hydroxylase enzyme in the kidney. Calcifediol is the major circulating
form of vitamin D. Calcitriol is the biologically active form of vitamin D. Calcitriol
stimulates the intestinal absorption of calcium and phosphate, the renal reabsorption of
filtered calcium, and the mobilization of calcium and phosphorus from bone. Vitamin D
deficiency results from lack of adequate sunlight coupled with a low dietary intake. An
infant requires only 30 minutes per week of total body sun exposure or 2 hours per
week of head exposure to maintain adequate vitamin D status. The young breast-fed
infant can acquire sufficient vitamin D from human milk if the mother's vitamin D
status is optimal. Otherwise, a vitamin D supplement is required to avoid risk of rickets.
In the United States, cow's milk and infant formulas are routinely supplemented with
vitamin D. Nutritional rickets may occur in older infants and children who are not
exposed to the sun or whose skin is deeply pigmented and who do not drink vitamin D
fortified milk. Vitamin D deficiency also occurs in fat malabsorption syndromes,
including small intestinal disease, cholestasis, and lymphatic obstruction. Use of P-450–
stimulating drugs may decrease hydroxylated vitamin D, which can also be decreased
by hepatic and renal disease and by inborn errors of metabolism. End-organ
unresponsiveness to calcitriol may also occur. The clinical effects of vitamin D
deficiency are osteomalacia (adults) or rickets (children), in which there is an
accumulation in bone of osteoid (matrix) with reduced calcification. Cartilage fails to
mature and calcify. The effects include craniotabes, rachitic rosary, pigeon breast,
bowed legs, delayed eruption of teeth and enamel defects, Harrison groove, scoliosis,
kyphosis, dwarfism, painful bones, fractures, anorexia, and weakness. X-ray findings
include cupping, fraying, and flaring of metaphyses; the loss of sharp definition
of bone trabeculae accounts for the general decrease in skeletal radiodensity. The
diagnosis is supported by characteristic radiologic abnormalities of the skeleton, low
serum phosphorus levels, high serum alkaline phosphatase levels, and high PTH levels.
The diagnosis can be confirmed by a low level of serum calcifediol. Rickets is treated
with 1600–5000 IU/d of vitamin D3 (1 IU = 0.25 µg). If this is poorly absorbed,
calcifediol, 2 µg/kg/d, or calcitriol, 0.05–0.2 µg/kg/d, is given. Renal osteodystrophy is
treated with calcitriol.

1046.- Con respecto a la vacunación con BCG se puede decir que:

a) Esta contraindicada en el recién nacido


b) Previene la infección por Mycobacterium tuberculosis
c) Reduce el riesgo de tuberculosis miliar y meníngea
d) Aumenta la protección inmunológica contra Mycobacterium avium
e) Se puede aplicar en niños con SIDA

Bacille Calmette-Guérin vaccine (BCG) consists of live attenuated Mycobacterium bovis.


It can be given any time after birth, sensitizes the vaccinated individual for 5–50 years,
and stimulates both B cell and T cell immune responses. BCG reduces the risk of
tuberculous meningitis and disseminated TB in pediatric populations by 50-
100%. Despite this, BCG appears to have had little epidemiologic impact on TB, with a
wide range of reported efficacy (0–80%).

1047.- Los anticuerpos más especificos en la cirrosis biliar primaria son:

a) Anticuerpos anti-acetilcolina
b) Anticuerpos antigliadina

MX-Biomedical Research Group / JD-MD Bioinformatics Labs 29


JD-MD Medicine and Technology
ENARM 11ª PARTE by JD-MD

c) Anticuerpos antinucleares
d) Anticuerpos anti-centromero
e) Anticuerpos anti-VHC

Primary biliary cirrhosis (PBC) is an autoimmune liver disease that generally affects
middle-aged women and is the most common chronic cholestatic liver disease in adults
in the United States. PBC is characterized by ongoing inflammatory destruction of the
interlobular and septal bile ducts that leads to chronic cholestasis and biliary cirrhosis,
with consequent complications such as portal hypertension and liver failure. Evidence
for an immunologic cause of PBC includes immunohistochemical data showing the
presence of activated T cells in areas of bile duct destruction, the presence of highly
specific autoantibodies reactive with antigens on the surface of biliary epithelial cells,
and the association of PBC with other disorders thought to be autoimmune in nature.
PBC may be discovered incidentally on routine blood testing but also can present with
pruritus or fatigue. The diagnosis should be considered in the setting of elevated serum
alkaline phosphatase, cholesterol, and immunoglobulin M (IgM) levels. The presence
of antimitochondrial antibody in serum is highly characteristic of the disease.
Ursodeoxycholic acid (UDCA) is the only medication of proven benefit for patients with
PBC; liver transplantation represents a life extending alternative for patients who have
end-stage PBC. It also is important to recognize and treat the complications of chronic
cholestasis, such as osteopenic bone disease, fat-soluble vitamin deficiency,
hypercholesterolemia, and steatorrhea. Survival models are used in determining the
prognosis of the disease as well as the timing of liver transplantation. AMAs have been
shown to be directed against the E2 component of the pyruvate dehydrogenase
complex (PDC-E2), the E2 unit of the branched-chain 2-oxo-acid dehydrogenase
complex (BCOADC-E2), and the E2 subunit of the 2-oxo-glutarate dehydrogenase
complex (OGDC-E2), as well as the E1 subunit of PDC and protein X. These enzymes all
are located on the inner mitochondrial membranes. At least one of these components
usually reacts with AMA in patients with PBC. The most frequent antigen against which
AMAs are directed is PDC-E2; PDC-E2 reacting antibodies are present in more than 90%
of PBC patients. AMAs are not the only PBC-specific autoantibodies. Antibodies
against the nuclear pore protein gp210, a transmembrane glycoprotein, are found
in 25% of patients who have AMA-positive PBC and in up to 50% of those who have
AMA-negative PBC. Their specificity for PBC when detected by immunoblotting is above
99%, and they seem to have prognostic importance. Antibodies against p62, a nuclear
pore glycoprotein, are found in about 25% of patients with PBC and are highly specific
for PBC. Anti-p62 antibodies and anti-gp210 antibodies appear to be mutually exclusive
in PBC patients. AMAs are present in serum in approximately 90% to 95% of patients
with PBC. The M2 antibody, which is directed against the pyruvate dehydrogenase
complex of the inner mitochondrial membrane, has a sensitivity rate of 98% and
specificity rate of 96% for the diagnosis of PBC. Other autoantibodies found in patients
with PBC are rheumatoid factor (70%), antismooth muscle antibody (66%), antithyroid
(antimicrosomal, antithyroglobulin) antibody (41%), and antinuclear antibody
(35%).

1048.- El tratamiento de elección en un paciente con un brote psicótico es:

a) Choque eléctrico
b) Haloperidol
c) Diazepam
d) Paroxetina
e) Fenitoina

An acutely agitated or psychotic patient can be calmed in a very short period of time.
For several years various protocols for rapid neuroleptization were promulgated;

MX-Biomedical Research Group / JD-MD Bioinformatics Labs 30


JD-MD Medicine and Technology
ENARM 11ª PARTE by JD-MD

however, it was never demonstrated that rapid relief of acute psychosis led to shorter
hospitalizations or that it was in the best interests of the patient to use this treatment.
A common practice in the United States is to administer high-potency
dopamine receptor antagonist (haloperidol), intramuscularly if necessary and
a benzodiazepine concurrently. This allows for a lower dose of the antipsychotic,
and a decreased likelihood of extrapyramidal adverse effects. Although there are as yet
no controlled studies regarding the use of the newer seroton-dopamine antagonists or
anticonvulsants in emergency room settings, their efficacy and relative lack of
extrapyramidal symptoms make them potentially useful alternatives.
Los antipsicóticos típicos tienen como mecanismo de acción el bloquear el receptor
post-sinaptico D2 de la dopamina. Sus usos cliniclos incluyen: tratamiento de la
esquizofrenia, psicósis asociada a otra enfermedad mental, tratamiento de síntomas
positivos (ej, alucinaciones, delusiones, thought broadcasting, etc).

1049.- El cáncer más frecuente de la piel es:

a) Carcinoma basocelular
b) Epitelioma espinocelular
c) Enfermedad de Bowen
d) Carcinoma espinoso
e) Melanoma

Basal cell carcinoma is the most common malignancy of the skin. Fortunately, a basal
cell epithelioma or carcinoma is almost never a metastasizing tumor, and the cure rate
can be close to 100% if these lesions are treated early and adequately. There are four
clinical types of basal cell carcinoma: (1) noduloulcerative, (2) pigmented, (3) fibrosing
(sclerosing, morphea-like), and (4) superficial. The noduloulcerative basal cell
carcinoma is the most common type. It begins as a small waxy nodule that enlarges
slowly over the years. A central depression usually forms that eventually progresses
into an ulcer surrounded by a pearly or waxy border. The surface of the nodular
component has a few telangiectatic vessels, which are highly characteristic. The
pigmented type is similar to the noduloulcerative form, with the addition of brown or

MX-Biomedical Research Group / JD-MD Bioinformatics Labs 31


JD-MD Medicine and Technology
ENARM 11ª PARTE by JD-MD

black pigmentation. The fibrosing type is extremely slow growing, is usually seen on
the face, and consists of a whitish, scarred plaque with an ill-defined border, which
rarely becomes ulcerated. This type is difficult to treat. The superficial form may be
single or multiple, is usually seen on the back and the chest, and is characterized by
slowly enlarging red, scaly areas that, on careful examination, reveal a nodular border
with telangiectatic vessels. A healed atrophic center may be present. Ulceration is
superficial when it develops.

1050.- En el aborto incompleto el tratamiento indicado es:

a) Inducto-conducción del trabajo de aborto


b) Legrado uterino instrumental
c) Administración de utero inhibidores
d) Aplicación de cerclaje
e) Unicamente reposo

El aborto incompleto es la retención de restos abortivos (habitualmente, restos


placentarios) en la cavidad uterina. Precisa que se realice un legrado uterino.

1051.- Caso clínico seriado: Un lactante de 9 meses presenta a la exploracion


física fontanela amplia, craneotabes, escleroticas azules y metafisis
ensanchadas. Primer enunciado: El diagnóstico es:

a) Cretinismo
b) Escorbuto
c) Beriberi
d) Raquitismo
e) Osteogénesis imperfecta

1052.- Segundo enunciado: El tratamiento adecuado para este lactante


consiste en administrarle:

a) 500 mg/día de vitamina C


b) 4000 a 5000 UI/día de vitamina D3
c) 10 mg/día de complejo B
d) 100 mg de gluconato de caldio/kg de peso/día
e) 6 a 8 mcg de tiroxina/kg de peso/día

La ostegenesis imperfecta (OI) es un trastorno del colágeno tipo I caracterizado por la


fragilidad ósea en huesos osteoporóticos; otros defectos que pueden aparecer son
escleróticas zules y articulaciones laxas. Este trastorno se manifiesta con distinta
intensidad y se han descrito cuatro variantes: 1) tipo I, el más común, clásico, más
leve, debido a un rasgo autosómico dominante en la osteogénesis imperfecta tardía,
llamada también enfermedad de Lobstein; aparece en la infancia y suele mejorar hacia
la adolescencia en la cual son más raras las fracturas; 2) tipo II, perinatal letal, tiene
tres subtipos. La forma recesiva se conoce como enfermedad de Vrolik, en la que
aparecen fracturas ya en la vida intrauterina por las contracciones uterinas apreciables
en las radiografías. There is no cure for OI. For severe nonlethal OI, active physical
rehabilitation in the early years allows children to attain a higher functional level than
does orthopedic management alone. Children with OI type I and some with type IV are
spontaneous ambulators. Children with type III and severe type IV benefit from long-leg
plastic braces, gait aids, and a program of swimming and conditioning. Severely
affected individuals require a wheelchair for community mobility but can acquire
transfer and self-care skills. Teens with OI may require psychologic support with body
image issues. Orthopedic management of OI is aimed at fracture management and

MX-Biomedical Research Group / JD-MD Bioinformatics Labs 32


JD-MD Medicine and Technology
ENARM 11ª PARTE by JD-MD

correction of deformity to enable function. Fractures should be promptly splinted or


cast; OI fractures heal well, and cast removal should be aimed at minimizing
immobilization osteoporosis. Correction of deformity of the long bones requires an
osteotomy procedure and placement of an intramedullary rod. Treatments with
calcium or fluoride supplements or calcitonin do not improve OI. Growth
hormone improves bone histologic characteristics in growth-responsive children
(usually types I and IV). Treatment with bisphosphonate drugs is effective in
improving mobility and decreasing symptoms in many patients. Intravenous
pamidronate or oral alendronate improves quality of life and inhibits bone
resorption, thus increasing bone mineralization; these agents decrease fractures and
bone pain, although the matrix still contains mutant type I collagen. The benefit for
trabecular bone appears most promising, with increased vertebral bone density and
height. The treatment effect may be independent of severity, mutation type, or age at
onset of therapy. The effect of bisphosphonates on development and on the
mechanical properties of cortical bone in long bones is under investigation.

1053.- El germen que con mayor frecuencia produce infeccion de las vias
urinarias en los pacientes diabeticos es:

a) Escherichia coli
b) Staphylococcus aureus
c) Staphylococcus epidermidis
d) Proteus mirabilis
e) Pseudomona aeruginosa

Individuals with DM have a greater frequency and severity of infection. The reasons for
this include incompletely defined abnormalities in cell-mediated immunity and
phagocyte function associated with hyperglycemia, as well as diminished
vascularization. Hyperglycemia aids the colonization and growth of a variety of
organisms (Candida and other fungal species). Many common infections are more
frequent and severe in the diabetic population, whereas several rare infections are
seen almost exclusively in the diabetic population. Examples of this latter category
includes rhinocerebral mucormycosis, emphysematous infections of the gall bladder
and urinary tract, and "malignant" or invasive otitis externa. Invasive otitis externa is
usually secondary to P. aeruginosa infection in the soft tissue surrounding the external
auditory canal, usually begins with pain and discharge, and may rapidly progress to
osteomyelitis and meningitis. These infections should be sought, in particular, in
patients presenting with hyperglycemic hyperosmolar state. Pneumonia, urinary tract
infections, and skin and soft tissue infections are all more common in the diabetic
population. In general, the organisms that cause pulmonary infections are similar to
those found in the nondiabetic population; however, gram-negative organisms, S.
aureus, and Mycobacterium tuberculosis are more frequent pathogens. Urinary tract
infections (either lower tract or pyelonephritis) are the result of common
bacterial agents such as Escherichia coli, though several yeast species
(Candida and Torulopsis glabrata) are commonly observed. Complications of
urinary tract infections include emphysematous pyelonephritis and emphysematous
cystitis. Bacteriuria occurs frequently in individuals with diabetic cystopathy.
Susceptibility to furunculosis, superficial candidal infections, and vulvovaginitis are
increased. Poor glycemic control is a common denominator in individuals with these
infections. Diabetic individuals have an increased rate of colonization of S. aureus in
the skin folds and nares. Diabetic patients also have a greater risk of postoperative
wound infections. Strict glycemic control reduces postoperative infections in diabetic
individuals undergoing coronary artery bypass grafting and should be the goal in all
diabetic patients with an infection.

MX-Biomedical Research Group / JD-MD Bioinformatics Labs 33


JD-MD Medicine and Technology
ENARM 11ª PARTE by JD-MD

1054.- Recien nacido prematuro con peso de 1,250 gramos y apgar de 6-7,
inicio a los pocos minutos de vida con quejido respiratorio, taquipnea, tiros
intercostales bajos, retraccion xifoidea, aleteo nasal y acrocianosis. El
diagnostico más probable es:

a) Neumonía in utero
b) Aspiración de meconio
c) Mala adaptación pulmonar
d) Enfermedad de la membrana hialina
e) Taquipnea transitoria del recien nacido

The respiratory distress syndrome (RDS) is the most common cause of death among
premature infants. It is caused by immaturity of the lungs and occurs in approximately
70% of infants weighing less than 1,500 g, but in less than 1% of those whose birth
weight is over 2,500 g. Males, infants of diabetic mothers, infants delivered by
cesarean section, and infants born to a mother who has had a previous child with RDS
have a particularly high incidence of this disorder. Asphyxia during labor or delivery
also seems to predispose the neonate to RDS. Conditions that may lessen the risk of
RDS include prolonged rupture of membranes, maternal drug addiction, and
intrauterine growth retardation. The primary abnormality in RDS appears to be
deficient synthesis or release of pulmonary surfactant. This material lines the alveoli of
the mature lung and lowers the surface tension required to keep the alveolus open
during expiration. When surfactant is deficient, diffuse alveolar atelectasis ensues,
resulting in pulmonary insufficiency. Surfactant production is normally very low in utero
until the third trimester of pregnancy; it then increases gradually until 33–36 weeks'
gestation, when there is an abrupt rise to near-term levels. Respiratory distress often
develops in infants born prior to this “surge” and does not improve until surfactant
production “turns on” at 48–72 hours after birth. Those who die during the acute stage
of respiratory insufficiency have profound, generalized atelectasis on autopsy. Often,
an eosinophilic material composed of fibrin, hemoglobin products, and cellular debris
coats the terminal bronchioles; this is the classically described “hyaline membrane,”
but it is not present in all cases, nor is it specific for RDS. Respiratory distress
syndrome produces characteristic clinical and radiographic findings. Tachypnea, nasal
flaring, and retractions are usually present within the first hour of life and are often
noted at birth. As the infant becomes progressively distressed, cyanosis is evident, and
an expiratory grunt or cry is audible. Auscultation reveals poor breath sounds
throughout the chest, often with fine, scattered crackles. The chest roentgenogram
shows a diffuse granular opacification of the lung fields, with prominent air
bronchograms; the heart border and diaphragms are often obscured. Blood gas
analysis demonstrates hypoxemia and, in severe cases, hypercapnia and acidosis.
Signs and symptoms usually worsen through the first 3 days of life, after which
improvement and recovery are the rule, unless complications such as patent ductus
arteriosus (PDA) or bronchopulmonary dysplasia have developed.

1055.- La prueba de Tensilón es de utilidad para establecer el diagnóstico de:

a) Neurastenia
b) Hipertiroidismo
c) Miastenia gravis
d) Botulismo
e) Oftalmoplejia progresiva externa

Parálisis bulbar asténica, miastenia genuina, miastenia grave o miastenia


seudoparalítica (Erb-Goldflam). Probablemente se trata de una profunda perturbación
del metabolismo muscular (disminución de acetilcolina o aumento de colinesterasa);

MX-Biomedical Research Group / JD-MD Bioinformatics Labs 34


JD-MD Medicine and Technology
ENARM 11ª PARTE by JD-MD

parece demostrada una defectuosa liberación presináptica de acetilcolina, con astenia


grave de los músculos. En las tres cuartas partes de los casos se ha encontrado
hipertrofia tímica y en algunos verdaderos timomas, y también se han publicado casos
de curación de la miastenia por extirpación del timo (50 por 100 de curaciones), que
indicarán un hipertimismo como concausa de la miastenia. Posiblemente se trata de un
bloqueo de los receptores en la placa motora de origen inmunológico. Se han
demostrado también anticuerpos antinucleares. Afecta, sobre todo, a mujeres entre
veinte y cuarenta años. Hay bastantes casos iniciados o agravados, otras veces
mejorados, en el embarazo. Consiste en una fatigabilidad extrema de varios grupos
musculares, principalmente de la cara, ojos, boca, faringe y laringe, que puede llegar a
la impotencia seudoparalítica. Los síntomas más llamativos son: ptosis palpebral, que
da un aire adormilado a la cara y obliga al paciente a andar con la cabeza echada hacia
atrás para compensar la ptosis; puede ser durante largos años (15 en un caso mío) la
única localización de la miastenia; estrabismo; parálisis facial; paresia de la lengua,
paladar, faringe, maseteros, cuerdas vocales, diafragma: por tanto, disartria, voz nasal,
reflujo de los líquidos por la nariz, disfonía y afonía, ageusia; puede haber también
paresia de los músculos del tronco y de los miembros; los esfínteres están siempre
respetados; no hay atrofia muscular o es sólo inicial. La pupila reacciona con típica
pereza (pupila miasténica) o normalmente en otros momentos. Lo típico de este
síndrome es que, después de un largo reposo, desaparecen las miastenias,
reapareciendo por el cansancio de los músculos respectivos y acentuándose a medida
que aumenta el ejercicio. Muchos enfermos advierten esta fatigabilidad al masticar. A
la exploración eléctrica existe la reacción miasténica (Jolly), pero no es positiva en
todos los casos y se presenta también en algunos enfermos de poliomielitis, etc. La
cronaxia es inestable. Los reflejos normales pero si se exploran repetidamente la
respuesta va siendo más débil. Curso progresivo, lento, con alternativas; aun cuando
hay casos de curación, suele terminar, generalmente, por la muerte. La juventud de los
enfermos, la íntima relación de la fatigabilidad y la paresia con el ejercicio, la ausencia
de lesiones vasculares, los posibles trastornos endocrinos (sobre todo el hipertiroidismo
y la comprobación de la hipertrofia del timo), la perturbación del metabolismo
muscular (Þ); las reacciones eléctricas, cuando son positivas; y la prueba del alivio de
los síntomas por la prostigmina, diferencian bien este síndrome de los bulbares.
También la prueba del Tensilón y la electromiografía proporcionan hallazgos típicos.
El síndrome de Lambert-Eaton es un cuadro miasténico que se presenta en el curso de
un carcinoma bronquial de células de avena y afecta a las extremidades inferiores —
simulando una paraparesia— más que a los nervios craneales.

1056.- El agente etiologico mas probable en un recien nacido de 3 semanas


de vida que presenta conjuntivitis y en cuya radiografia de torax se
encuentra infiltrado intersticial es:

a) Haemophilus influenzae
b) Chalmydia trachomatis
c) Staphylococcus aureus
d) Candida albicans
e) Streptococcus pneumoniae

Perinatal infection by Chlamydia trachomatis is generally acquired during passage


through an infected birth canal; however, it has been documented after cesarean
delivery, especially after premature rupture of amniotic membranes. In such infants
exposed to C. trachomatis, risk for conjunctivitis is 25% to 50%, and risk for pneumonia
is 5% to 20%. The nasopharynx, the most common site of colonization, is positive in up
to 70% of infected infants; 8% to 22% of colonized infants experience pneumonia.
Nasopharyngeal colonization can persist for 3 years or longer. C. trachomatis is the
most common cause of ophthalmia neonatorum, which is the major clinical

MX-Biomedical Research Group / JD-MD Bioinformatics Labs 35


JD-MD Medicine and Technology
ENARM 11ª PARTE by JD-MD

manifestation of neonatal chlamydial infection. The usual incubation period is 5 to 14


days after birth, but symptoms can occur earlier after premature rupture of
membranes or as late as 6 weeks after birth. At least 50% of infants with chlamydial
conjunctivitis have concurrent infection in the nasopharynx. Typically, a watery ocular
discharge appears which becomes progressively more purulent. The eyelids swell, and
the conjunctivae become injected. If the condition is untreated, lymphoid follicles and a
pseudomembranous conjunctivitis can develop and persist for weeks or months. Scars
and pannus formation are rare. Mild or subclinical infection can persist in some cases
for years. The neovascularization of the cornea resulting from repeated infection in
classic trachoma does not occur with neonatal disease. The conjunctivitis must be
distinguished from that produced by pyogenic bacteria, particularly Neisseria
gonorrhoeae. Gonococcal ophthalmia usually occurs earlier, around 2 to 5 days after
birth, and is usually more rapidly progressive than ophthalmia due to C. trachomatis.
Neonatal conjunctivitis due to N. gonorrhoeae can be indistinguishable from that due to
C. trachomatis, however, on the basis of clinical findings and time of onset. Afebrile
pneumonia caused by C. trachomatis in infancy occurs characteristically between 4 and
11 weeks of age; it has been noted at 2 weeks but not beyond 4 months. Symptoms of
nasal obstruction and staccato cough without fever gradually worsen over a week or
more. Characteristically, the infant has been symptomatic for 3 or more weeks before
presentation. Most infants are only moderately ill and are afebrile. Physical findings
include tachypnea and rales but not wheezing. About 50% of affected infants have
a history or evidence of conjunctivitis; a similar proportion have middle ear
abnormalities. Laboratory findings include hyperinflation with symmetrical
interstitial infiltrates on chest radiography, peripheral eosinophilia (>400
cells/mm3), and increased levels of serum immunoglobulins. Follow-up evaluation of a
small cohort of children who had C. trachomatis pneumonia in infancy has shown an
increased prevalence of chronic cough and abnormal lung function when compared
with age-matched controls.

1057.- Un preescolar de 5 años inicio su padecimiento hace una semana con


febricula y malestar general, posteriormente aparecieron lesiones
maculopapulares eritematosas en las regiones gluteas y las extremidades
inferiores; más tarde se extendieron al torax y se agregaron flogosis en las
rodillas y dolor abominal difuso. El diagnostico más probable es:

a) Artritis reumatoide juvenil


b) Lupus eritematosis sistemico
c) Purpura de Henoch-Schönlein
d) Fiebre reumatica
e) Artritis reactiva

Henoch-Schönlein purpura, the most common small vessel vasculitis syndrome in


children, occurs primarily in males aged 2–7 years. Occurrence is highest in the spring
and fall. Two thirds of affected children have a history of upper respiratory infection in
the preceding 1–3 weeks. Henoch-Schönlein purpura is characterized by vasculitis of
the small vessels, particularly those of the skin, gastrointestinal tract, and kidney. The
most common and earliest symptom is palpable purpura, which results from
extravasation of erythrocytes into the tissue surrounding the involved venules. Serum
IgA immune complexes are thought to trigger a leukocytoclastic vasculitis. Immune
complexes are found in the kidney, intestine, and skin. Suspected though not proved
inciting antigens include group A b-hemolytic streptococci and other bacteria, viruses,
drugs, foods, and insect bites. The skin rash is often urticarial initially and progresses to
a macular-papular appearance, which transforms into a diagnostic symmetric purpuric
rash distributed on the ankles, buttocks, and elbows. Purpuric areas of a few
millimeters in diameter are present and may progress to form larger hemorrhages

MX-Biomedical Research Group / JD-MD Bioinformatics Labs 36


JD-MD Medicine and Technology
ENARM 11ª PARTE by JD-MD

(palpable purpura). Petechial lesions may occur, but most hemorrhages are slightly
larger. The rash usually begins on the lower extremities and buttocks, but the entire
body may be involved. New lesions can continue to appear for 2–4 weeks.
Approximately two thirds of patients develop migratory polyarthralgias or polyarthritis,
primarily of the ankles and knees. Edema of the hands, feet, scalp, and periorbital
regions may occur. Abdominal colic—due to hemorrhage and edema primarily of the
small intestine—occurs in about 50% of those affected. Between 25% and 50% of those
affected develop renal involvement, with hematuria, proteinuria, or nephrotic
syndrome. Some children develop hypertension. Hematuria alone is never the
presenting complaint for Henoch-Schönlein purpura but usually manifests in the second
to third week of illness. Renal involvement occurs more commonly in males than in
females and in older patients than in younger ones. Testicular torsion may occur.
Neurologic symptoms are possible. Generally, treatment is unwarranted. Corticosteroid
therapy may provide symptomatic relief for severe gastrointestinal or joint
manifestations but does not alter skin or renal manifestations. Aspirin is useful for the
pain associated with arthritis. If culture for group A b-hemolytic streptococci is positive
or if the ASO titer is elevated, penicillin should be given in full doses for 10 days.

1058.- El tratamiento adecuado para un niño de dos años que presenta


criptorquidia unilateral consiste en:

a) Esperar a que cumpla seis años y practivar orquidopexia


b) Administrar testosterona
c) Practicar orquiectomia de inmediato
d) Administrar gonadotropina corionica
e) Practicar la orquidopexia de inmediato

Cryptorchism (undescended testis) is a common disorder in children. It may be


unilateral or bilateral and may be classified as ectopic or true cryptorchism.
Approximately 3% of term male newborns have an undescended testis at birth, with a
higher proportion among premature infants. In over 50% of these patients, the testes
descend by the third month; by age 1 year, 80% of all undescended testes are in the
scrotum. Further descent may occur through puberty, the latter perhaps stimulated by
endogenous gonadotropins. Plasma testosterone concentrations may be obtained after
hCG stimulation to confirm the presence or absence of abdominal testes. The child with
bilaterally undescended testes should be evaluated for sex chromosome abnormalities;
evaluation should include consideration of the possibility that the child is a virilized
female. The current recommendation in the treatment of cryptorchism is for
surgical orchidopexy performed by an experienced surgeon when the child is
age 1 year. Gonadotropin therapy (hCG) has been used in the treatment for
cryptorchism but is not generally successful. Hormone treatment can be useful in the
identification and descent of retractile testes or to evaluate for the presence of
testicular tissue. Various treatment regimens have been used, ranging from 250 to
1000 IU given twice weekly for 5 weeks, and will generally cause descent of retractile
testes. Androgen treatment (eg, depot testosterone) is indicated as replacement
therapy in the male child who lacks functional testes beyond the normal age of
puberty.

1059.- Cual es la conversion metabólica de la vitamina D en la piel:

a) Transformacion de 7-dehidrocolesterol en vitamina D3


b) Transformacion de la vitamina D a 25-hidroxivitamina D
c) Conversion de 25-hidroxivitamina D a 1,25 hidroxivitamina D
d) Conversion de ergocalciferol a colecalciferol
e) Transformacion de vitamina D2 a 25-hidroxivitamina D
MX-Biomedical Research Group / JD-MD Bioinformatics Labs 37
JD-MD Medicine and Technology
ENARM 11ª PARTE by JD-MD

The active transport of Ca2+ and PO43- from the intestine is increased by a metabolite
of vitamin D. The term "vitamin D" is used to refer to a group of closely related sterols
produced by the action of ultraviolet light on certain provitamins. Vitamin D3, which
is also called cholecalciferol, is produced in the skin of mammals from 7-
dehydrocholesterol by the action of sunlight. The reaction involves the rapid
formation of previtamin D3, which is then converted more slowly to vitamin D3
(cholecalciferol). In the liver, vitamin D3 is converted to 25-hydroxycholecalciferol
(calcidiol, 25-OHD3). The 25-hydroxycholecalciferol is converted in the cells of the
proximal tubules of the kidneys to the more active metabolite 1,25-
dihydroxycholecalciferol, which is also called calcitriol or 1,25-(OH)2D3. Vitamin D
deficiency causes defective calcification of bone matrix and the disease called rickets
in children and osteomalacia in adults. The full-blown condition in children is
characterized by weakness and bowing of weight-bearing bones, dental defects, and
hypocalcemia. In adults, the condition is less obvious. It used to be most commonly due
to inadequate exposure to the sun in smoggy cities, but now it is more commonly due
to inadequate intake of the provitamins on which the sun acts in the skin.

1060.- Fisiopatología de la diabetes mellitus tipo 2:

a) Resistencia a la insulina mas hiperinsulinismo


b) Disminución de la resistencia insulinica
c) Disminución de la glucogenolisis
d) Aumento de la producción hepatica de glucogeno
e) Disminución de la gluconeogenesis

DM2 is a complex of polygenic disorders. Certain families show an autosomal


dominance. The genetic defects differ and many mutations are known. One is the gene
on chromosome 7, which code for glucokinase. Identical twins show almost absolute
concordance in development of DM2. The much more frequent type 2 diabetes is
the result of insulin resistance and b-cell defects. Type 2 diabetes also occurs in
younger persons, especially in persons with a high fat-low muscle mass. A strong
genetic element is always present, but inactivity and stress (an inactive life style with a
low endurance capacity) seems to be involved in the development of type 2 diabetes.
Lack of exercise predisposes one to obesity, a condition that greatly decreases insulin
sensitivity of the target cells (adipocytes, heart and skeletal muscle tissues). Reduced
glucose combustion creates hyperglycaemia. The hyperglycaemia elicits insulin
secretion from defective b-cells in some patients, resulting in raised serum [insulin].
Since insulin is present, the acute complications such as ketonaemia and metabolic
acidaemia are rare in these patients. The high serum insulin may further down-regulate
the activity of their insulin receptors. The insulin secreted in DM2 patients does
not increase the uptake of glucose as in normal persons. Many DM2 patients
need much more insulin for a given test effects than DM1 patients and healthy people.
An inactive life style for years, with redundant food intake, seems to be involved in the
development of DM2 in persons with a genetic predisposition. Lack of regular physical
activity with development of overweight, increases the incidence if DM2. The impaired
glucose tolerance is demonstrated by a glucose tolerance test. The insulin secretion
is abnormal in patients with DM2, although they typically possess half of their b-
cell mass at autopsy. The destroyed b-cells is filled with amyloid material (islet amyloid
polypeptide, IAPP). IAPP is a possible antagonist to insulin, and explain some
cases of insulin resistance. Many older patients with DM2 have no symptoms, but a
routine examination reveals glucosuria or a raised blood glucose. Other patients are
tired, have minor genital infections or sugar spots on their underwear. Some patients
present with established late-complications such as retinopathy (blindness),
nephropathy, arteriosclerotic disorders (cerebrovascular insults, myocardial infarction,

MX-Biomedical Research Group / JD-MD Bioinformatics Labs 38


JD-MD Medicine and Technology
ENARM 11ª PARTE by JD-MD

intermittent claudication, gangrene), susceptibility to infection or neuropathy. DM2 can


be caused, theoretically, by 1) b-cell defects including genetic defects, resulting
in abnormal insulin production, or by 2) target cell defects including receptor
failure. The possible defective sites in 1 and 2 have one common denominator. They
are all key proteins (hormone, receptors and transporters). Muscular activity is required
to stimulate the normal production of key proteins. DM2 relates to inactive life style.
The basic problem is therefore possibly a genetically and activity dependent defect in
key protein production in the cell interior. Actually, a genetic defect has just been
demonstrated at certain steps of insulin action in a subset of patients of late-onset
DM2.

1061.- Neoplasia gastrica más frecuente:

a) Adenocarcinoma
b) Carcinoma indiferenciado
c) Linfoma
d) Leiomiosarcoma
e) Carcinoma anaplasico

Gastric adenocarcinoma is the second most common malignancy worldwide.


Lymphoma is the second most common malignancy encountered in the stomach.
Primary gastric lymphomas account for 3–5% of gastric neoplasms. The large majority
of these lymphomas are B cell non-Hodgkin's lymphomas of the diffuse, large cell type.
Gastric lymphoma is the most common extranodal lymphoma, accounting for 20–24%
of primary extranodal lymphomas. Less frequent primary gastric malignancies include
carcinoid tumors and gastrointestinal stromal cell tumors (GISTs). The occurrence of
carcinoids in the stomach is rare; approximately 95% of all carcinoid tumors occur in
the rectum, appendix, and small intestine. GISTs represent the largest category of
nonepithelial neoplasms of the gastrointestinal tract.

1062.- Urgencia cardíaca frecuente en los estados de tirotoxicosis:

a) Fibrilación ventricular
b) Fibrilación auricular con respuesta ventricular rápida
c) Hipertensión arterial
d) Insuficiencia cardiaca
e) Taquicardia ventricular

Sinus tachycardia at rest and during sleep as well as during exercise is the
most consistent rhythm disturbance in patients with thyrotoxicosis. About
5% to 10% of patients have atrial fibrillation; most are elderly, but it can occur in
young adults. The ventricular rate in atrial fibrillation is often rapid as a result
of the increased rate of conduction of the electrical impulses through the
atrioventricular node. Most patients with atrial fibrillation have the arrhythmia for
less than 4 to 8 weeks before the diagnosis of thyrotoxicosis is made. In the absence of
evidence for chronicity, most patients spontaneously revert to sinus rhythm within 8 to
12 weeks after antithyroid treatment is begun. Spontaneous reversion to sinus rhythm
after treatment of the thyrotoxicosis is less common in elderly patients with chronic
atrial fibrillation or patients who have underlying coronary or other heart disease or
anatomic abnormalities of the mitral valve or left atrium. The development of
persistent atrial fibrillation poses the potential for systemic embolization and stroke.
The risk of embolism seems small and is limited largely to older patients with
coexistent heart disease. The value of anticoagulation is controversial; it is discussed in
more detail later. Atrial flutter and other supraventricular tachyarrhythmias (including

MX-Biomedical Research Group / JD-MD Bioinformatics Labs 39


JD-MD Medicine and Technology
ENARM 11ª PARTE by JD-MD

paroxysmal atrial tachycardia) are uncommon in patients with thyrotoxicosis.


Ventricular premature contractions and ventricular fibrillation are rare.

1063.- La function que se debe monitorizar en casos de intoxicación por


paracetamol es la:

a) Renal
b) Cardiaca
c) Hepatica
d) Pulmonar
e) Intestinal

Acetaminophen (paracetamol) is a common analgesic found in many nonprescription


and prescription products. After absorption, it is metabolized mainly by glucuronidation
and sulfation, with a small fraction metabolized via the P450 mixed-function oxidase
system to a highly toxic reactive intermediate. This toxic intermediate is normally
detoxified by cellular glutathione. With acute acetaminophen overdose (> 140 mg/kg,
or 7 g in an average adult), hepatocellular glutathione is rapidly depleted and
the reactive intermediate attacks other cell proteins, causing necrosis.
Patients with enhanced P450 activity, such as chronic alcoholics and patients taking
anticonvulsants, are at increased risk of developing hepatotoxicity. Hepatic toxicity
may also occur after chronic accidental overuse of acetaminophen - eg, as a
result of taking two or three acetaminophen-containing products concurrently or
intentionally exceeding the recommended maximum dose of 4 g/d. Shortly after
ingestion, patients may have nausea or vomiting, but there are usually no other signs
of toxicity until 24-48 hours after ingestion, when hepatic aminotransferase levels
begin to increase. With severe poisoning, fulminant hepatic necrosis may occur,
resulting in jaundice, hepatic encephalopathy, renal failure, and death. Rarely, massive
ingestion (eg, serum levels over 400-500 mg/L) can cause acute coma, hypotension,
and metabolic acidosis unrelated to hepatic injury. The diagnosis of severe poisoning
after acute overdose is based on measurement of the serum acetaminophen level.
Ingestion of sustained-release products or coingestion of an anticholinergic agent,
salicylate, or opioid drug may cause delayed elevation of serum levels and may render
the nomogram useless. The nomogram is not useful after chronic overdose. Administer
activated charcoal if it can be given within 1-2 hours of the ingestion. Although
charcoal may interfere with absorption of the oral antidote acetylcysteine, this is not
considered clinically significant. If the serum acetaminophen level is above the toxic
line on the nomogram, begin treatment with a loading dose of N-acetylcysteine, 140
mg/kg orally, followed by 70 mg/kg every 4 hours. Dilute the solution to 5% with water,
juice, or soda. If vomiting interferes with oral N-acetylcysteine administration, consider
giving the antidote intravenously. The most widely used oral N-acetylcysteine protocol
in the United States calls for 72 hours of treatment. However, other regimens have
demonstrated equivalent success with 20-48 hours of treatment. The San Francisco
Bay Area Poison Center recommends treatment until 36 hours after ingestion, at which
time N-acetylcysteine can be discontinued if liver transaminases are normal. Treatment
with N-acetylcysteine is most effective if started within 8-10 hours after ingestion. If the
precise time of ingestion is unknown or if the patient is at higher risk of hepatotoxicity
(eg, alcoholic, liver disease, chronic use of P450-inducing drugs), then use a lower
threshold for initiation of N-acetylcysteine (ie, the lower nomogram line; in some case
reports, a level of 100 mg/L at 4 hours was suggested in very high-risk patients). N-
acetylcysteine may also be given intravenously; this is the preferred method in Europe
and Canada, and a parenteral formulation (Acetadote) was recently approved by the
Food and Drug Administration (FDA) for use in the United States. The conventional oral
formulation may also be given intravenously using a micropore filter and a slow rate of
infusion. Call a regional poison control center or medical toxicologist for assistance.

MX-Biomedical Research Group / JD-MD Bioinformatics Labs 40


JD-MD Medicine and Technology
ENARM 11ª PARTE by JD-MD

1064.- El azul de metileno esta indicado en el tratamiento de la intoxicación


por:

a) Anilina
b) Morfina
c) Dextrometorfan
d) Cianuro
e) Fenotiazidas

El azul de metileno es una sustancia cristalina verde-azulada que se utiliza como tinte
histológico e indicador de laboratorio. También se emplea en el tratamiento de la
intoxicación por cianuro y en la metahemoglobinemia.

1065.- El tratamiento de la enfermedad celiaca consiste en:

a) Suprimir el gluten de la dieta


b) Proporcionar una dieta libre de lactosa
c) Administrar colestiramina
d) Indicar una dieta rica en trigliceridos
e) Administrar enzimas pancreaticas

La enfermedad celiaca es un error congénito del metabolismo caracterizado por la


incapacidad para hidrolizar los péptidos del gluten. La enfermedad afecta a adultos y
niños pequeños, y sus síntomas son distensión abdominal, vómitos, diarrea, pérdida de
masa muscular y letargia extrema. La mayoría de los pacientes responde bien a
una dieta sin gluten rica en proteínas y calorías.

1066.- Caso clinico seriado: Una paciente de 22 años, gesta 1, que cursa con
embarazo de 37 semanas de evolución, presenta perdida del conocimiento
posterior a crisis convulsivas tonicoclonicas, presion arterial de 170/120
mmHg, frecuencia cardiaca de 100 por minuto, reflejos osteoteninosos
aumentads, frecuencia cardiaca fetal de 130 por minuto y edema importante
en los miembros inferiores, no se aprecian signos de trabajo de parto ni
modificaciones cervicales. Primer enunciado: El diagnostico más probable es:

a) Eclampsia
b) Preeclampsia severa
c) Crisis epilepticas tipo gran mal
d) Crisis hipertensiva
e) Hipertension inducida por el embarazo

1067.- Segundo enunciado: Una posible complicación en esta paciente es el


síndrome de Hellp que consiste en:

a) Edema, hipertensión y albuminuria de mas de 3 grs


b) Elevación de las enzimas hepaticas y hemolisis
c) Hemolisis, plaquetopenia y elevación de las enzimas hepaticas
d) Anemia, plaquetopenia y hemolisis
e) Hemolisis, plaquetopenia e hipertension arterial

Eclampsia occurs in 0.2–0.5% of all deliveries, with occurrence being influenced by the
same factors as in preeclampsia. In rare instances, eclampsia develops before 20
weeks' gestation. About 75% of eclamptic seizures occur before delivery. About 50% of
postpartum eclamptic seizures occur in the first 48 hours after delivery, but they may

MX-Biomedical Research Group / JD-MD Bioinformatics Labs 41


JD-MD Medicine and Technology
ENARM 11ª PARTE by JD-MD

occur as late as 6 weeks postpartum. The pathogenesis of eclamptic seizures is poorly


understood. Seizures have been attributed to platelet thrombi, hypoxia due to localized
vasoconstriction, and foci of hemorrhage in the cortex. There is also a mistaken
tendency to equate eclampsia with hypertensive encephalopathy. There is a poor
correlation between occurrence of seizures and severity of hypertension. Seizures may
occur with insignificant blood pressure elevations that are only slightly higher than
readings recorded 24 hours previously. The hallmarks of hypertensive encephalopathy
(retinal hemorrhages, exudates, and papilledema) are very infrequent in eclampsia,
where funduscopic changes are minimal. There is usually no aura preceding the
seizure, and the patient may have one, two, or many seizures. Unconsciousness lasts
for a variable period of time. The patient hyperventilates after the tonic-clonic seizure
to compensate for the respiratory and lactic acidosis that develops during the apneic
phase. Fever is rare but is a poor prognostic sign. Seizure-induced complications may
include tongue biting, broken bones, head trauma, or aspiration. Pulmonary edema and
retinal detachment have also been noted following seizures.

1068.- El uso de corticoides esta indicado en los niños que padecen:

a) Síndrome nefritico
b) Asma
c) Hepatitis B
d) Artritis séptica
e) Meningitis viral

Management of status asthmaticus includes early supplemental oxygen, a


frequently or continuously administered b2-adrenergic agent (albuterol, terbutaline), a
nebulized anticholinergic agent (ipratropium), and a corticosteroid. Delivery of b2-
adrenergic agents may be achieved by either nebulization or metered-dose inhaler
(MDI) with spacer or face masks. Subcutaneous epinephrine may be indicated in
children with severe bronchospasm who are unable to tolerate nebulization treatment.
Intravenous (IV) terbutaline should be considered in children unresponsive to nebulized
b2-adrenergic treatment. A corticosteroid should be administered early, orally or
parenterally. Incremental benefit from the addition of methylxanthine (aminophylline)
appears to be offset by the increased incidence of adverse effects. Intravenous fluid
should be administered in limited quantity and monitored carefully to prevent
overhydration and pulmonary edema. Antibiotics are not routinely indicated but may
be considered in cases of suspected bacterial, mycoplasma, or Chlamydia pneumoniae
pneumonia. Adjunctive therapy for patients refractory to b-agonists and
corticosteroids includes IV magnesium sulfate, helium-oxygen (heliox), and ketamine.
Although there is insufficient evidence to recommend the routine use of magnesium
sulfate, it may be considered in severe airway obstruction with impending respiratory
failure. Heliox is a mixture of 60–80% helium and 20–40% oxygen. Because of its low
density (one third that of room air), it reduces airway resistance by converting
turbulent to laminar flow, thereby reducing work of breathing. The exact role of heliox
in status asthmaticus is unknown. Limited data suggest it may be a temporizing
measure that may avert the need for intubation and mechanical ventilation.

1069.- A que nivel actua la aldosterona:

a) Tubulo proximal
b) Membrana glomerular
c) Asa de Henle
d) Tubulo contorneado distal
e) Tubulo colector

MX-Biomedical Research Group / JD-MD Bioinformatics Labs 42


JD-MD Medicine and Technology
ENARM 11ª PARTE by JD-MD

Los principales sitios de acción de la aldosterona son el túbulo distal y la región cortical
de los colectores, donde promueve la reabsorción de sodio. La aldosterona se elabora
en la zona glomerulosa de la corteza suprarrenal. Es el mineralocorticoide más potente
producido en esta glandula. La tasa de secreción de aldosterona es cien veces inferior
a la del cortisol y su aclaramiento plasmático mucho más rápido.

1070.- A que nivel se lleva la mayor reabsorción de sodio y agua:

a) Túbulo contorneado proximal


b) Porción ascendente del asa de Henle
c) Túbulo colector
d) Túbulo contorneado distal
e) Arteriola aferente

El túbulo contorneado proximal resorbe alrededor del 70% del agua filtrada utilizando
como impulsor la fuerza osmótica transtubular, generada por la absorción de solutos.
Cuando el equilibrio de sodio corporal es el adecuado, la reabsorción de sodio y agua
aumenta en paralelo.

1071.- El antiparasitario cuya acción consiste en inhibir a la colinesterasa y


ocasionar la despolarización de la placa neuromuscular por medio de su
actividad nicotinica es:

a) Metronidazol
b) Ornidazol
c) Pamoato de pirantel
d) Piperacina
e) Pirimetamina

Pyrantel pamoate (Antiminth, Combantrin): depolarizing neuromuscular


blocker, wich causes parálisis of nematodos.
Pyrymethamine: inhibits dehydrofolate reductase. Uses (plus sulfadoxine) vs
primarily active against erythocytic forms of malaria, some activity vs primary
Plasmodium infection in the liver, pyrimethamine is used with sulfadiazine for
treatment of toxoplasmosis.
Metronidazole (Flagyl): Forms cytotoxic producs via reductive bioactivation of its nitro
group, causes oxidative damage to DNA of trophozoite. Most effective drug against
invasive form of Entamoeba histolytica, drug of choice for giardiasis, trichomoniasis,
Gardnerella vaginalis infections, and infections caused by anaerobic bacteria

1072.- La estimulación de los receptores beta-1 por acción de la dopamina


produce:

a) Inotropismo y cronotropismo negativos


b) Inotropismo positivo y cronotropismo negativo
c) Inotropismo y cronotropismo positivos
d) Disminución de la resistencia vascular periferica
e) Activación de la respuesta vagal

Dopamine uses: short-term use in cardiac decompensation secondary to decreased


contractility; increases organ perfusion (at low dose). Positive inotropic agent with
dose dose-related response. 2-10 mcg/kg/min B-effects (increases cardiac output
and renal perfusion). 10-20 mcg/kg/min B-effects (periferal vasocontriction, pressor). >
20 mcg/kg/min peripheral and renal vasoconstriction.

MX-Biomedical Research Group / JD-MD Bioinformatics Labs 43


JD-MD Medicine and Technology
ENARM 11ª PARTE by JD-MD

1073.- La digoxina es util para el tratamiento de insuficiencia cardíaca


cuando se acompaña de:

a) Arritmia sinusal
b) Extrasistoles ventriculares
c) Bloqueo auriculoventricular
d) Fibrilación auricular
e) Taquicardia sinusal

Glycosides of digitalis, which have been used in the treatment of heart failure for more
than 200 years, are the most frequently used inotropic agents and the only oral
positive inotropic preparation approved for treatment of heart failure. Digitalis
glycosides, of which digoxin if the agent most commonly used, relieve symptoms by
improving cardiac performance through increased myocardial contractility, improved
LV function, and increased cardiac output and renal perfusion. Neurohumoral
modulating actions have also been reported. Several studies suggest that treatment
with digitalis decreases plasma renin activity, attenuates sympathetic drive, reduces
plasma norepinephrine levels, and improves baroreceptor sensitivity. These effects are
seen as playing an important role in treating heart failure. The beneficial effect of
digitalis glycosides in patients with heart failure complicated by the
occurrence of atrial fibrillation has been well documented and is generally
well accepted by most clinicians. The results of several studies indicate that in
patients with heart failure, digitalis does exert sustained beneficial hemodynamic
effects accompanied by improvement in both clinical status and exercise tolerance.
(Those who did not benefit from treatment with digitalis had milder degrees of heart
failure or evidence of primarily diastolic dysfunction.) Because some evidence suggests
that use of digitalis in suspected or confirmed MI may be associated with adverse
outcome, it is appropriate to consider alternatives to treatment with digoxin for
patients with MI and heart failure.

1074.- En los casos de incompatibilidad por sistema ABO, es común observar


la prueba de Coombs:

a) Directa negativa
b) Directa positiva
c) Indirecta positiva
d) Indirecta negativa
e) Directa e indirecta positivas

La enfermedad hemolítica del recién nacido, o eritroblastosis fetal, es una enfermedad


del feto y del recién nacido, debida a la incompatibilidad sanguínea entre la madre y el
feto. La sensibilización por embarazo se produce cuando el feto hereda un antígeno
paterno ausente en la madre, que atraviesa la placenta durante el parto, estimulando
la producción de aloanticuerpos maternos. Otras veces, la sensibilización se produce
con anterioridad al embarazo, por la inyección o transfusión de sangre portadora de
antígenos ausentes en la madre. En ambos casos, los anticuerpos IgG de
sensibilización atraviesan la placenta, se unen a los antígenos de los eritrocitos fetales
y provocan su hemólisis. La sensibilización puede deberse a antígenos de los sistemas
ABO, Rhesus u otros sistemas de grupos sanguíneos. En la actualidad la EHRN por
incompatibilidad Rh es mucho menos frecuente que por ABO u otros. La madre Rh (D)
negativa se sensibiliza al antígeno D y forma anti-D por transfusión con sangre Rh
positiva o por un embarazo previo con un feto que ha heredado el antígeno D paterno.
Si, una vez sensibilizada, vuelve a embarazarse con un feto Rh positivo, los hematíes
fetales pueden entrar en la circulación materna, originando una respuesta inmune
secundaria en la madre, que formará anticuerpos anti-D, IgG. El anticuerpo IgG

MX-Biomedical Research Group / JD-MD Bioinformatics Labs 44


JD-MD Medicine and Technology
ENARM 11ª PARTE by JD-MD

sintetizado por la madre atraviesa la placenta, se une a los hematíes fetales e induce
su hemólisis. Clínicamente el feto puede estar ligera o gravemente afecto, según la
cantidad de anticuerpo y la capacidad de la eritropoyesis fetal para compensar la
hemólisis. Si la hemólisis puede ser compensada por el feto, este llegará a término sin
grandes problemas, dado que el exceso de bilirrubina es metabolizado por la madre. Si
la hemólisis es muy grave, el feto sufrirá anemia severa, insuficiencia cardiaca y puede
morir intraútero con grandes edemas; es el denominado hydrops fetalis. El
diagnóstico anteparto puede establecerse mediante la prueba de Coombs,
indirecta en la madre, y con espectofotometría del líquido amniótico, que
proporciona información sobre el grado de afectación fetal. El tratamiento
consistirá en transfusiones intraútero, exanguinotransfusión o fototerapia, según la
afectación fetal. Recientemente se han empleado con éxito las inmunoglobulinas
intravenosas.

1075.- Cantidad máxima de aire que puede expulsarse después de una


inspiración máxima:

a) Volumen corriente
b) Volumen residual
c) Capacidad residual funcional
d) Capacidad vital
e) Capacidad pulmonar total

1076.- Para el tratamiento de la encefalopatia hepatica crónica esta indicado:

a) Furosemida
b) Diazepam
c) Espironolactona
d) Lactulosa
e) Eritromicina

Hepatic encephalopathy is a state of disordered central nervous system function


resulting from failure of the liver to detoxify noxious agents of gut origin because of
hepatocellular dysfunction and portosystemic shunting. The clinical spectrum ranges
from day-night reversal and mild intellectual impairment to coma. Patients with
minimal hepatic encephalopathy have no recognizable clinical symptoms but
MX-Biomedical Research Group / JD-MD Bioinformatics Labs 45
JD-MD Medicine and Technology
ENARM 11ª PARTE by JD-MD

demonstrate mild cognitive and psychomotor deficits on standardized tests. Ammonia


is the most readily identified and measurable toxin but is not solely responsible for the
disturbed mental status. Pathogenic factors may include production of false
neurotransmitters, increased sensitivity of central nervous system neurons to the
inhibitory neurotransmitter g-aminobutyric acid (GABA), an increase in circulating
levels of endogenous benzodiazepines, decreased activity of urea-cycle enzymes due
to zinc deficiency, decreased brain levels of myoinositol, deposition of manganese in
the basal ganglia, and swelling of astrocytes in the brain. Bleeding into the intestinal
tract may significantly increase the amount of protein in the bowel and precipitate
rapid development of encephalopathy. Other precipitants include constipation,
alkalosis, and potassium deficiency induced by diuretics, opioids, hypnotics, and
sedatives; medications containing ammonium or amino compounds; paracentesis with
attendant hypovolemia; hepatic or systemic infection; and portosystemic shunts
(including TIPS). Dietary protein should be withheld during acute episodes if the patient
cannot eat. When the patient resumes oral intake, protein intake should be 60-80 g/d
as tolerated; vegetable protein is better tolerated than meat protein. Gastrointestinal
bleeding should be controlled and blood purged from the gastrointestinal tract. This
can be accomplished with 120 mL of magnesium citrate by mouth or nasogastric tube
every 3-4 hours until the stool is free of gross blood, or by administration of
lactulose. The value of treating patients with minimal hepatic encephalopathy is
uncertain. Lactulose, a nonabsorbable synthetic disaccharide syrup, is digested by
bacteria in the colon to short-chain fatty acids, resulting in acidification of colon
contents. Lactulose also leads to a change in bowel flora so that fewer
ammonia-forming organisms are present. When given orally, the initial dose of
lactulose for acute hepatic encephalopathy is 30 mL three or four times daily. The dose
should then be titrated so that two or three soft stools per day are produced. When
rectal use is indicated because of the patient's inability to take medicines orally, the
dose is 300 mL of lactulose in 700 mL of saline or sorbitol as a retention enema for 30-
60 minutes; it may be repeated every 4-6 hours. Lactilol is a less sweet disaccharide
alternative available as a powder in some countries. The ammonia-producing intestinal
flora may also be controlled with neomycin sulfate, 0.5-1 g orally every 6 or 12 hours
for 7 days. Side effects of neomycin include diarrhea, malabsorption, superinfection,
ototoxicity, and nephrotoxicity, usually only after prolonged use. Alternative antibiotics
are vancomycin, 1 g orally twice daily, or metronidazole, 250 mg orally three times
daily. Patients who do not respond to lactulose alone may improve with a 1-week
course of an antibiotic in addition to lactulose. Opioids and sedatives metabolized or
excreted by the liver are avoided. If agitation is marked, oxazepam, 10-30 mg, which is
not metabolized by the liver, may be given cautiously by mouth or by nasogastric tube.
Zinc deficiency should be corrected, if present, with oral zinc sulfate, 600 mg/d in
divided doses. There is limited evidence that eradication of Helicobacter pylori, which
generates ammonia in the stomach, may improve encephalopathy. Sodium benzoate,
10 g daily, and ornithine aspartate, 9 g three times daily, may lower blood ammonia
levels, but there is less experience with these drugs than with lactulose. The
benzodiazepine competitive antagonist flumazenil is effective in about 30% of patients
with severe hepatic encephalopathy, but the drug is short-acting and intravenous
administration is required. Use of special dietary supplements enriched with branched-
chain amino acids is usually unnecessary except in occasional patients who are
intolerant of standard protein supplements. Treatment by modulating the gut flora with
prebiotic and probiotic agents is under study.

1077.- Una lactante de ocho meses presenta un cuadro de 24 horas de


evolucion con fiebre de 39.2 °c, vomito y rechazo al alimento. Se encuentra
somnolienta e irritable con desviacion de la mirada hacia arriba y
movimientos anormales de las manos y los pies; rigidez de la nuca e

MX-Biomedical Research Group / JD-MD Bioinformatics Labs 46


JD-MD Medicine and Technology
ENARM 11ª PARTE by JD-MD

hiperreflexia osteotendinosa. Para confirmar el diagnostico se debe


practicar:

a) Biometría hematica completa, examen general de orina y ultrasonido


transfontanelar
b) Biometría hemática completa, punción lumbar y cultivo
c) Biometría hemática completa, examen de orina y radiografia de tórax
d) Radiografia de craneo, hemocultivo y tomografia craneal
e) Biometría hemática, tomografia del craneo y radiografia de tórax

Bacterial infections of the CNS may present acutely (symptoms evolving rapidly over 1–
24 hours), subacutely (symptoms evolving over 1–7 days), or chronically (symptoms
evolving over more than 1 week). Diffuse bacterial infections involve the
leptomeninges, superficial cortical structures, and blood vessels. Although the term
“meningitis” is used to describe these infections, it should not be forgotten that the
brain parenchyma is also inflamed and that blood vessel walls may be infiltrated by
inflammatory cells that result in endothelial cell injury, vessel stenosis, and secondary
ischemia and infarction. While awaiting the results of diagnostic tests, the physician
should start broad-spectrum antibiotic coverage as noted below. After specific
organisms are identified, antibiotic therapy can be tailored based on antibiotic
sensitivity patterns. Bacterial meningitis in children under age 3 months is treated
initially with cefotaxime (or ceftriaxone if the child is over age 1 month) and ampicillin;
the latter agent is used to treat Listeria and enterococci infections, which rarely affect
older children. Children over age 3 months are given ceftriaxone, cefotaxime, or
ampicillin plus chloramphenicol. If Streptococcus pneumoniae cannot be ruled out by
the initial Gram stain, vancomycin or rifampin are added until cultures are reported,
because penicillin-resistant pneumococci are common in the United States. Therapy
may be narrowed when organism sensitivity allows. Duration of therapy is 7 days for
meningococcal infections, 10 days for H influenzae or pneumococcal infection, and 14–
21 days for other organisms. Slow clinical response or the occurrence of complications
may prolong the need for therapy. Although therapy for 7 days has proved successful
in many children with H influenzae infection, it cannot be recommended without further
study if steroids are also used. Neuroimaging with CT and MRI scans may be helpful in
demonstrating the presence of brain abscess, meningeal inflammation, or secondary
problems such as venous and arterial infarctions, hemorrhages, and subdural effusions
when these are expected. In addition, these procedures may identify sinus or other
focal infections in the head or neck region that are related to the CNS infection. CT
scanning may demonstrate bony abnormalities, such as basilar fractures. EEGs may be
helpful in the assessment of patients who have had seizures at the time of
presentation. The changes are often nonspecific and characterized by generalized
slowing. In some instances, such as herpes simplex virus infection, focal
electronegative activity may be seen early in the course and may be one of the earliest
laboratory abnormalities to suggest the diagnosis. EEGs may also show focal slowing
over regions of abscesses. Unusual but characteristic electroencephalographic patterns
are seen in some patients with subacute sclerosing panencephalitis.

1078.- El mecanismo principal por el que las bacterias alcanzan el


parenquima renal y producen pielonefritis en el niño es:

a) Vaciamiento vesical frecuente


b) Peristalsis ureteral
c) Recurrencia de infecciones
d) Diseminación hematogena
e) Reflujo vesiculouretral

MX-Biomedical Research Group / JD-MD Bioinformatics Labs 47


JD-MD Medicine and Technology
ENARM 11ª PARTE by JD-MD

Pediatric urinary tract infection (UTI) may involve the urethra, bladder, or kidney.
Outside the clinical setting of systemic perinatal infection or prematurity, UTIs in
infants raise the possibility of urinary tract abnormalities. Boys less than 3 months of
age are more susceptible than girls. During the first year of life, circumcised males are
more likely to experience UTI due to a urinary tract abnormality than are females and
uncircumcised males. Older boys with a first infection should also be examined for
urinary tract abnormalities. A more conservative, watchful approach may be taken with
older girls, especially if they are sexually active or have poor personal hygiene and no
clinical features that arouse suspicion of significant urinary tract disease (eg, enuresis
or short stature). At least 8% of girls and 2% of boys will have a UTI in childhood. It is
particularly important to detect these infections in young children, since many children
less than 5 years of age with febrile UTIs have pyelonephritis. In this age group, the
infection commonly leads to renal scarring, especially in those less than a year of age.
Focal renal scarring may be a risk factor for hypertension and renal disease later in life.
The most common organisms causing UTI are E coli (> 80%), Klebsiella, Proteus
(increaed in males), enterococcus, all normal fecal flora, and, infrequently,
Staphylococcus. In the absence of bacteremia, enteric organisms are presumed to gain
access to the urinary tract via the urethra. The likelihood of infection is increased
with abnormalities of the urinary tract, poor perineal hygiene or infection,
sexual activity, instrumentation, or dysfunctional voiding. Normal voiding
serves to safeguard against bacterial contamination of the urethra
developing into infection. Newborns may exhibit fever or hypothermia, poor
feeding, jaundice, failure to thrive, or sepsis. Infants may have unexplained fever,
irritability, or foul-smelling urine. Preschool children may have abdominal pain,
vomiting, strong-smelling urine, fever, enuresis, or frequency, dysuria, or urgency.
School-age children are often afebrile, but usually have the classic signs of UTI, such as
enuresis, frequency, dysuria, and urgency. Costovertebral angle tenderness may be
elicited in cases of pyelonephritis. It is very rare for children with bacterial UTI to
present with hemorrhagic cystitis, which is more common with viral infections. Urinary
tract infection is the most common occult bacterial cause of unexplained fever in
infants less than 2 years of age. Sick children should be treated presumptively when
the laboratory findings are suggestive. Initial therapy should be based on prior
antibiotic use, location of the infection (eg, kidney, bladder), and the organism and its
drug sensitivities. Uncomplicated urethritis or cystitis can be treated with a single oral
antibiotic that the patient has not used recently. Amoxacillin, trimethoprim-
sulfamethoxazole (TMP-SMX), or a later generation cephalosporin can be used as initial
therapy. Choice of antibiotic should also take into account community bacterial
resistance patterns. TMP-SMX has generally yielded higher cure rates when compared
with amoxacillin. TMP-SMX and cephalosporins are not active against enterococcus.
Enterococcus might be suspected when a Gram stain is performed. Duration of therapy
should be 7 to 10 days. Short course, high dose therapy should not be used in children.
The choice of antibiotic should be reviewed when culture and sensitivity results are
available. In toxic, dehydrated children who cannot receive oral therapy, especially if
there is suspected pyelonephritis, hospitalization should be considered. Many of these
children will be bacteremic (~5% of children less than 2 years of age). A non-toxic,
older child with suspected pyelonephritis need not be admitted to the hospital (as long
as compliance is not in question), but antibiotic coverage should be broad (ampicillin or
cephalosporin plus gentamicin) until the bacteria's sensitivity to antibiotics is known.
Antibiotic dosage must be adjusted for patients with associated acute or chronic renal
failure. If the organism is sensitive to the antibiotic chosen and the child improves
within 2 days, no test of cure is needed. If symptoms persist, bacterial sensitivity
should be checked, the child reexamined, and a repeat urine culture obtained.
Persistent symptoms and bacteriuria indicate one of the following: 1) wrong choice of
antibiotic; 2) development of bacterial resistance; 3) superinfection with a different
organism; or 4) a significant anatomic abnormality in the urinary tract.

MX-Biomedical Research Group / JD-MD Bioinformatics Labs 48


JD-MD Medicine and Technology
ENARM 11ª PARTE by JD-MD

1079.- La complicación hematologíca que más frecuentemente presentan los


hijos de madres diabeticas es la:

a) Policitemia
b) Plaquetopenia
c) Leucopenia
d) Hiperleucocitosis
e) Enfermedad hemorragica del recién nacido

Los lactantes con un riesgo especial de padecer policitemia son los lactantes PEG a
término y postérmino, los niños de madres diabéticas, los lactantes con pinzamiento
tardío del cordón y los que presentan hipertiroidismo neonatal, síndrome
adrenogenital, trisomía 21, síndrome de transfusión gemelar (receptor) y síndrome de
Beckwith-Wiedemann. En algunos lactantes, la policitemia puede reflejar una
compensación por pedíodos prolongados de hipoxia feral debida a insuficiencia
placentaria; estos lactantes tienen niveles de eritropoyetina más altos en el
nacimiento.

1080.- Caso clinico seriado: Hombre de 32 años, desde hace tres meses
presenta debilidad muscular; se fatiga facilmente y hace algunos días noto la
aparicion de manchas hiperpigmentadas en las rodillas, el cuello y en las
areas expuestas al sol. Sus examenes de laboratorio muestran: sodio 123
mEq/l y potasio 5.3 mEq/l. Primer enunciado: el diagnostico probable es:

a) Hipercortisolismo
b) Feocromocitoma
c) Insuficiencia suprarrenal
d) Aldosteronismo primario
e) Hipotiroidismo

Addison's disease is an uncommon disorder caused by destruction or dysfunction of the


adrenal cortices. It is characterized by chronic deficiency of cortisol, aldosterone, and
adrenal androgens and causes skin pigmentation that can be subtle or strikingly dark.
Volume and sodium depletion and potassium excess eventually occur in primary
adrenal failure. In contrast, if chronic adrenal insufficiency is secondary to pituitary
failure (atrophy, necrosis, tumor), mineralocorticoid production (controlled by the renin-
angiotensin system) persists and hyperkalemia is not present. Furthermore, if ACTH is
not elevated, skin pigmentary changes are not encountered. Autoimmune destruction
of the adrenals is the most common cause of Addison's disease in the United States
(accounting for about 80% of spontaneous cases). Tuberculosis was formerly a leading
cause of Addison's disease. The association is now relatively rare in the United States
but common where tuberculosis is more prevalent. The symptoms may include
weakness and fatigability, weight loss, myalgias, arthralgias, fever, anorexia, nausea
and vomiting, anxiety, and mental irritability. Some of these symptoms may be due to
high serum levels of IL-6. Pigmentary changes consist of diffuse tanning over
nonexposed as well as exposed parts or multiple freckles; hyperpigmentation is
especially prominent over the knuckles, elbows, knees, and posterior neck and in
palmar creases and nail beds. Nipples and areolas tend to darken. The skin in pressure
areas such as the belt or brassiere lines and the buttocks also darkens. New scars are
pigmented. Some patients have associated vitiligo (10%). Emotional changes are
common. Hypoglycemia, when present, may worsen the patient's weakness and
mental functioning, rarely leading to coma. Manifestations of other autoimmune
disease (see above) may be present. Patients tend to be hypotensive and orthostatic;
about 90% have systolic blood pressures under 110 mm Hg; blood pressure over 130

MX-Biomedical Research Group / JD-MD Bioinformatics Labs 49


JD-MD Medicine and Technology
ENARM 11ª PARTE by JD-MD

mm Hg is rare. Other findings may include a small heart, hyperplasia of lymphoid


tissues, and scant axillary and pubic hair (especially in women). The white count
usually shows moderate neutropenia, lymphocytosis, and a total eosinophil count over
300/mcL. Among patients with chronic Addison's disease, the serum sodium is usually
low (90%) while the potassium is elevated (65%). Patients with diarrhea may not be
hyperkalemic. Fasting blood glucose may be low. Hypercalcemia may be present.
Young men with idiopathic Addison's disease are screened for adrenoleukodystrophy
by determining plasma very long-chain fatty acid levels; affected patients have high
levels. Replacement therapy should include a combination of glucocorticoids and
mineralocorticoids. In mild cases, hydrocortisone alone may be adequate.
Hydrocortisone is the drug of choice. Most addisonian patients are well maintained on
15-25 mg of hydrocortisone orally daily in two divided doses, two-thirds in the morning
and one-third in the late afternoon or early evening. Some patients respond better to
prednisone in a dosage of about 2-3 mg in the morning and 1-2 mg in the evening.
Adjustments in dosage are made according to the clinical response. A proper dose
usually results in a normal differential white count. Many patients, however, do not
obtain sufficient salt-retaining effect and require fludrocortisone supplementation or
extra dietary salt. Fludrocortisone acetate has a potent sodium-retaining effect. The
dosage is 0.05-0.3 mg orally daily or every other day. In the presence of postural
hypotension, hyponatremia, or hyperkalemia, the dosage is increased. Similarly, in
patients with fatigue, elevated plasma renin activity indicates the need for a higher
replacement dose of fludrocortisone. If edema, hypokalemia, or hypertension ensues,
the dose is decreased. DHEA is given to some women with adrenal insufficiency.
Women taking DHEA 50 mg orally each morning have experienced an improvement in
their overall sense of well-being, mood, and sexuality. Because over-the-counter
preparations of DHEA have variable potencies, it is best to have the pharmacy
formulate this with pharmaceutical-grade DHEA.

1081.- Segundo enunciado: estudio para apoyar el diagnóstico:

a) Determinación de TSH
b) Determinación de ácido vanilmandelico
c) Determinación de cortisol en el plasma
d) Determinación de renina
e) Prueba de dexametasona

Low plasma cortisol (< 3 mcg/dL) at 8 AM is diagnostic, especially if


accompanied by simultaneous elevation of the plasma ACTH level (usually > 200
pg/mL). The diagnosis is made by a simplified cosyntropin stimulation test, which is
performed as follows: (1) Synthetic ACTH1-24 (cosyntropin), 0.25 mg, is given
parenterally. (2) Serum is obtained for cortisol between 30 and 60 minutes after
cosyntropin is administered. Normally, serum cortisol rises to at least 20 mcg/dL. For
patients receiving glucocorticoid treatment, hydrocortisone must not be given for at
least 8 hours before the test. Other glucocorticoids (eg, prednisone, dexamethasone)
do not interfere with specific assays for cortisol. Serum DHEA levels are under 1000
ng/mL in 100% of patients with Addison's disease and a serum DHEA above 1000
ng/mL excludes the diagnosis However, serum DHEA levels below 1000 ng/mL are not
helpful, since about 15% of the general population have such low DHEA levels,
particularly children and elderly individuals. Antiadrenal antibodies are found in the
serum in about 50% of cases of autoimmune Addison's disease. Antibodies to thyroid
(45%) and other tissues may be present. Elevated plasma renin activity indicates the
presence of depleted intravascular volume and the need for higher doses of
fludrocortisone replacement.

1082.- Tratamiento urgente de la crisis corticosuprarrenal:

MX-Biomedical Research Group / JD-MD Bioinformatics Labs 50


JD-MD Medicine and Technology
ENARM 11ª PARTE by JD-MD

a) Reposición de líquidos y ACTH


b) Glucanato de calcio y aldosterona
c) Reposición de líquidos y vasopresina
d) Solución mixta y somatostatina
e) Solución salina e hidrocortisona

Acute adrenal insufficiency is an emergency caused by insufficient cortisol. Crisis may


occur in the course of chronic treated insufficiency, or it may be the presenting
manifestation of adrenal insufficiency. Acute adrenal crisis is more commonly seen in
primary adrenal insufficiency (Addison's disease) than in disorders of the pituitary
gland causing secondary adrenocortical hypofunction. Adrenal crisis may occur in the
following situations: (1) following stress, eg, trauma, surgery, infection, or prolonged
fasting in a patient with latent insufficiency; (2) following sudden withdrawal of
adrenocortical hormone in a patient with chronic insufficiency or in a patient with
temporary insufficiency due to suppression by exogenous glucocorticoids or megestrol;
(3) following bilateral adrenalectomy or removal of a functioning adrenal tumor that
had suppressed the other adrenal; (4) following sudden destruction of the pituitary
gland (pituitary necrosis), or when thyroid is given to a patient with hypoadrenalism;
and (5) following injury to both adrenals by trauma, hemorrhage, anticoagulant
therapy, thrombosis, infection, or, rarely, metastatic carcinoma. The patient complains
of headache, lassitude, nausea and vomiting, abdominal pain, and often diarrhea.
Confusion or coma may be present. Fever may be 40.6°C or more. The blood pressure
is low. Patients with preexisting type 1 diabetes may present with recurrent
hypoglycemia and reduced insulin requirements. Other signs may include cyanosis,
dehydration, skin hyperpigmentation, and sparse axillary hair (if hypogonadism is also
present). Meningococcemia may be associated with purpura and adrenal insufficiency
secondary to adrenal infarction (Waterhouse-Friderichsen syndrome). The eosinophil
count may be high. Hyponatremia or hyperkalemia (or both) are usually present.
Hypoglycemia is frequent. Hypercalcemia may be present. Blood, sputum, or urine
culture may be positive if bacterial infection is the precipitating cause of the crisis. The
diagnosis is made by a simplified cosyntropin stimulation test, which is performed as
follows: (1) Synthetic ACTH1-24 (cosyntropin), 0.25 mg, is given parenterally. (2)
Serum is obtained for cortisol between 30 and 60 minutes after cosyntropin is
administered. Normally, serum cortisol rises to at least 20 mcg/dL. For patients
receiving glucocorticoid treatment, hydrocortisone must not be given for at least 8
hours before the test. Other glucocorticoids (eg, prednisone, dexamethasone) do not
interfere with specific assays for cortisol. Plasma ACTH is markedly elevated if the
patient has primary adrenal disease (generally > 200 pg/mL). If the diagnosis is
suspected, draw a blood sample for cortisol determination and treat with
hydrocortisone, 100-300 mg intravenously, and saline immediately, without waiting for
the results. Thereafter, give hydrocortisone phosphate or hydrocortisone sodium
succinate, 100 mg intravenously immediately, and continue intravenous infusions of
50-100 mg every 6 hours for the first day. Give the same amount every 8 hours on the
second day and then adjust the dosage in view of the clinical picture. Since bacterial
infection frequently precipitates acute adrenal crisis, broad-spectrum antibiotics should
be administered empirically while waiting for the results of initial cultures.
Hypoglycemia should be vigorously treated while serum electrolytes, BUN, and
creatinine are monitored. When the patient is able to take food by mouth, give oral
hydrocortisone, 10-20 mg every 6 hours, and reduce dosage to maintenance levels as
needed. Most patients ultimately require hydrocortisone twice daily (AM, 10-20 mg; PM,
5-10 mg). Mineralocorticoid therapy is not needed when large amounts of
hydrocortisone are being given, but as the dose is reduced it is usually necessary to
add fludrocortisone acetate, 0.05-0.2 mg daily. Some patients never require
fludrocortisone or become edematous at doses of more than 0.05 mg once or twice

MX-Biomedical Research Group / JD-MD Bioinformatics Labs 51


JD-MD Medicine and Technology
ENARM 11ª PARTE by JD-MD

weekly. Once the crisis has passed, the patient must be investigated to assess the
degree of permanent adrenal insufficiency and to establish the cause if possible.

1083.- Caso clínico seriado: Un recien nacido primogenito de 3 semanas de


vida extrauterina, presenta vómitos de contenido gástrico posprandial
inmediato. Primer enunciado: El diagnóstico más probable es:

a) Enfermedad por reflujo


b) Atresia intestinal
c) Malrotación intestinal
d) Hipertrofia congenita del piloro
e) Membrana duodenal

The cause of postnatal pyloric circular muscle hypertrophy leading to gastric outlet
obstruction is not known. The incidence is 1–8:1000 births, with a 4:1 male
predominance and a positive family history present in 13% of patients. Recent studies
suggest that erythromycin therapy may be associated with development of pyloric
stenosis in infants under 30 days. Vomiting usually begins between ages 2 and 4 weeks
and rapidly becomes projectile after every feeding; it starts at birth in about 10% of
cases. Onset of symptoms may be delayed in premature infants. The vomitus is rarely
bilious but may be blood-streaked. The infant is hungry and nurses avidly.
Constipation, dehydration, weight loss, fretfulness, and finally apathy occur. The upper
abdomen may be distended after feeding, and prominent gastric peristaltic waves from
left to right may be seen. An olive-sized mass can be felt on deep palpation in the right
upper abdomen, especially after the child has vomited. Hypochloremic alkalosis with
potassium depletion occurs. Hemoconcentration is reflected by elevated hemoglobin
and hematocrit values. Elevated unconjugated bilirubin occurs in 2–5% of cases.

1084.- Segundo enunciado: Para confirmar el diagnóstico se debe practicar:

a) Endoscopia
b) Serie esofagogastroduodenal
c) Radiografía simple de abdomen
d) Colon por enema
e) Ultrasonido

An upper gastrointestinal series reveals delay in gastric emptying and an


elongated narrowed pyloric channel with a double tract of barium. Many
infants have some delay in gastric emptying due to pylorospasm. This by itself is
insufficient to make a diagnosis of pyloric stenosis. The enlarged pyloric muscle causes
characteristic semilunar impressions on the gastric antrum. Ultrasonography shows
a hypoechoic ring with a hyperdense center. Thickness of circular muscle is
greater than 4 mm in pyloric stenosis.

1085.- Tercer enunciado: El tratamiento quirugico más adecuado para este


paciente es:

a) Funduplicatura de Nissen
b) Piloromiotomia
c) Resección intestinal
d) Colostomia
e) Gastrostomia

Pyloromyotomy is the treatment of choice and consists of incision down to the mucosa
along the pyloric length. The procedure can be performed laparoscopically. Prior to

MX-Biomedical Research Group / JD-MD Bioinformatics Labs 52


JD-MD Medicine and Technology
ENARM 11ª PARTE by JD-MD

surgery, it is imperative to repair hydration and electrolyte abnormalities even if it


takes 24–48 hours. The outlook is excellent following surgery. Patients often vomit
postoperatively as a consequence of gastritis, esophagitis, or associated
gastroesophageal reflux. The postoperative barium x-ray remains abnormal despite
relief of symptoms.

1086.- Cual es el mejor tratamiento en la brucelosis humana:

a) Ciprofloxacino
b) Rifampicina
c) Tetraciclina
d) Tetraciclina + estreptomicina
e) TMP-SMX

Brucellosis (also called undulant fever, Mediterranean fever, Malta fever) is an infection
that causes abortion in domestic animals. It is caused by one of six species of Brucella
coccobacilli. It may occasionally be transmitted to humans, in whom the disease could
be acute or chronic with ongoing fever and constitutional symptoms without localized
findings. After a 2- to 8-week incubation period, affected patients present with a
spectrum of disease that varies from an asymptomatic form to a severe illness with
bacteremia. Affected patients typically present with fever, sweats, headaches, malaise,
and weight loss. If undiagnosed, these symptoms may persist for months to a year.
When symptoms last for more than 12 months without any localization, the disease is
classified as chronic. Physical examination, though normal in most cases, may reveal
bilateral diffuse lymphadenopathy, splenomegaly, and hepatomegaly in 20%-30% of
patients. In the localized form, the infection may affect virtually any organ system. The
osteoarticular, gastrointestinal, genitourinary, and cardiovascular are among the more
common affected systems. These forms have signs and symptoms related to the
affected organ system. An infectious discitis with adjacent vertebral bony osteomyelitis
typically in the lumbar area, is the most common manifestation of localized
osteoarticular disease. Sacroilitis is also characteristic of the musculoskeletal form of
the infection. Infective endocarditis occurs in < 2% of brucellosis cases. Patients with
chronic brucellosis may have long periods of no symptoms followed by the intermittent
recurrence of fever, chills, myalgias, and nonspecific symptoms. This form of relapsing
brucellosis may persist for decades and is often refractory to antimicrobial therapy and
is frequently associated with multiple or large calcific lesions in the liver and spleen.
Many other common illnesses can mimic the most common clinical presentation of
brucellosis.
Treatment of acute brucellosis first choice:
Adults: Doxycycline, 100 mg orally twice daily, PLUS rifampin, 600-900 mg
orally daily for 4-6 weeks
Children: Doxycycline,2 2 mg/kg orally twice daily, PLUS rifampin, 15 mg/kg orally daily
for 4-6 weeks
Treatment of acute brucellosis second choice:
Adults: Doxycycline, 100 mg orally twice daily for 4-6 weeks, PLUS streptomycin, 1 g IM
daily for the first 15 days
Children: TMP/SMX,3 2.5 mg/kg orally of TMP component 4 times daily, PLUS rifampin,
15 mg/kg orally daily for 4-6 weeks
Treatment of Endocarditis-Meningitis-Spondylitis-Localized Forms first choice
Adults: Doxycycline, 100 mg orally twice daily, PLUS rifampin, 600-900 mg orally daily
for 6 mo, PLUS streptomycin, 1 g IM daily for the first 2-3 weeks
Children: Doxycycline,2 2 mg/kg orally twice daily, PLUS rifampin, 15 mg/kg orally daily
for 6 mo, PLUS streptomycin, 10 mg/kg IM twice daily for the first 7-14 days
Treatment of Endocarditis-Meningitis-Spondylitis-Localized Forms second
choice

MX-Biomedical Research Group / JD-MD Bioinformatics Labs 53


JD-MD Medicine and Technology
ENARM 11ª PARTE by JD-MD

TMP/SMX,3 2.5 mg/kg orally of TMP component 4 times daily, PLUS rifampin, 15 mg/kg
orally daily for 6 mo, PLUS gentamicin, 1.5-2 mg/kg (IV or IM) in three daily doses for
the the first 7-14 days

1087.- Lugar donde actua la paratohormona:

a) Medula ósea, eritrocitos y plaquetas


b) Sistema nervioso central, tejido musclar y plaquetas
c) Tejido renal, tejido óseo y mucosa intestinal
d) Músculo cardíaco, cerebro y músculo estriado
e) Eritrocito, tejido hepatico y estomago

1088.- En la actualidad se ha abandonado la monoterapia en el tratamiento


de la brucelosis, indique el tratamiento deprimera linea combinado:

a) Ampicilina + amikacina por 2 semanas


b) TMP-SMX por 2 semanas
c) Doxiciclina + estreptomicina por 2 semanas
d) Doxiciclina + rifampicina durante 4 semanas a 8 semanas
e) Rifampicina + ampicilina por 3 semanas

Treatment of acute brucellosis first choice:


Adults: Doxycycline, 100 mg orally twice daily, PLUS rifampin, 600-900 mg
orally daily for 4-6 weeks
Children: Doxycycline,2 2 mg/kg orally twice daily, PLUS rifampin, 15 mg/kg orally daily
for 4-6 weeks
Treatment of acute brucellosis second choice:
Adults: Doxycycline, 100 mg orally twice daily for 4-6 weeks, PLUS streptomycin, 1 g IM
daily for the first 15 days
Children: TMP/SMX,3 2.5 mg/kg orally of TMP component 4 times daily, PLUS rifampin,
15 mg/kg orally daily for 4-6 weeks

1089.- El antimicrobiano más útil para el tratamiento de la disenteria por


Shigella es:

a) Amikacina
b) Trimetoprim-sulfametoxazol
c) Gentamicina
d) Eritromicina
e) Cefutoxima

Enteritis caused by Shigella species may be watery (Shigella sonnei, Shigella boydii) or
dysenteric (Shigella dysenteriae, Shigella flexneri). Risks include ingestion of fecally
contaminated food or water and contact with infected individuals. Definitive diagnosis
requires microbiologic isolation and identification of Shigella species or molecular
evidence of infection. Early in the course of disease, when bacteria are present in the
small intestine, patients develop acute, watery diarrhea; fever; and abdominal pain.
Patients may become toxemic and fever may reach as high as 104 °F. Later in the
course of disease, the primary site of infection is the colon. In this phase, fever
continues, but is usually less pronounced. The pain that is present is usually in the
lower abdominal quadrants. Stools become dysenteric, consisting of a mixture of
neutrophils, blood, mucus, and debris. Frequent, small-volume or fractionated stools
may occur, and tenesmus is often present. Patients experience pain upon rectal
examination. Colonoscopy discloses hyperemic and friable-to-ulcerated colonic
mucosa. Patients with acute diarrhea, which may be watery to dysenteric; fever;
MX-Biomedical Research Group / JD-MD Bioinformatics Labs 54
JD-MD Medicine and Technology
ENARM 11ª PARTE by JD-MD

abdominal pain; and systemic symptomatology/toxemia may have shigellosis. A history


of exposure to individuals with shigellosis, travel to endemic areas, and exposure to a
high-risk population, such as persons in a custodial-care facility, should raise the index
of suspicion. The presence of leukocytes in the stool, although supportive, is by no
means definitive for shigellosis. Fecal leukocytes may be present in the stools of
patients with other bacterial enteritides, amoebic dysentery, pseudomembranous
colitis, and noninfectious disease, such as inflammatory-bowel disease. The definitive
diagnosis requires the microbiologic identification of a Shigella species. Shigellae are
particularly susceptible to some environmental changes, and they die rapidly in
transport. Therefore, it is imperative to rapidly transport the stool of patients suspected
of having shigellosis to the laboratory. This is especially important for patients in the
latter stages of disease, in whom the number of shigellae in the stool are relatively few.
Fluid and electrolyte replacements are necessary for patients with dehydration. In most
instances, this is readily accomplished by oral rehydration. Unlike in many other
bacterial enteritides, antibiotic therapy is important in the treatment of shigellosis.
Antibiotic therapy limits the clinical course of the disease, may decrease the likelihood
of intestinal complications, and decreases the fecal excretion of viable pathogenic
organisms, which in turn diminishes transmission. Fluoroquinolones are the
treatment of choice for adults. TMP/SMX is the treatment of choice for
children. Alternatives are ampicillin, chloramphenicol, and nalidixic acid. In areas of
known resistance to TMP/SMX, such as parts of Southeast Asia, Africa, and South
America, quinolones should be used for adults, and one of the above mentioned
alternatives for children with shigellosis. When available, the antimicrobial-
susceptibility profile should guide therapy. Antimotility agents, such as diphenoxylate,
should not be used. The inhibition of diarrhea increases the contact between the
intestinal mucosa and the pathogenic organisms and their toxins and may cause more
fulminant disease.

1090.- Pneumocystis carinii es un patogeno oportunista colocado dentro del


reino de los hongos, se considera que el fármaco más adecuado para atacar
todas las formas de esta micosis es:

a) Miconazol
b) TMP-SMX
c) Praziquantel
d) Amikacina
e) Anfotericina B

Pneumocystis carinii, when examined using molecular techniques, most closely


resembles a fungus. Stains of either bronchoalveolar-lavage (BAL) or transbronchial-
biopsy samples yield a diagnosis in > 90% of patients and should be considered the
gold standard in diagnosis. BAL with transbronchial biopsy increases diagnostic yield to
~ 100%. P. carinii has not yet been cultured in vitro. Polymerase chain reaction (PCR)
(especially on sputum) increases sensitivity but reduces specificity. The prophylactic
use of aerosolized pentamidine reduces the sensitivity of sputum and bronchoscopic
samples. The primary treatment of moderate to severe pulmonary or
extrapulmonary infection caused by P carinii remains the combination of
trimethoprim (TMP) and sulfamethoxazole (SMX), either orally or
intravenously (IV). Several medications (atovaquone, trimetrexate, and pentamidine)
and combination regimens (dapsone + TMP and clindamycin + primaquine) probably
afford nearly equal efficacy to TMP-SMX in mild to moderate PCP and may be better
tolerated in specific populations of patients.

MX-Biomedical Research Group / JD-MD Bioinformatics Labs 55


JD-MD Medicine and Technology
ENARM 11ª PARTE by JD-MD

1091.- En la enfermedad de Anisen debida a Micobacterium leprae, un bacilo


intracelular obligado acidorresistente, se recomienda como tratamiento
antimicrobiano:

a) Dapsona + rifampicina
b) Isoniazida + etambutol
c) Ciprofloxacina + etambutol
d) Isonicida + ciprofloxacina
e) Ceftrixona + amikacina

The manifestations of leprosy involve the skin, upper respiratory system, peripheral
nerves, and, in men, testes. The peripheral neuropathy that is seen in leprosy results in
impaired sensation of fine touch, temperature, and pain, whereas proprioception and
vibratory sensation are intact. Such loss of sensation leads to recurrent trauma and
ulceration in the extremities. For example, loss of sensation in the feet can lead to
chronic nonhealing ulcers, especially at the metatarsal heads. Large nerve trunks may
also be affected. A common nerve trunk that is affected in leprosy is the ulnar nerve at
the elbow, which results in clawing of the fourth and fifth digits of the hand, dorsal
interosseus muscle atrophy, and loss of sensation in the hand along the ulnar nerve
distribution. The lepromatous or multibacillary form of leprosy is characterized by
symmetric skin nodules, plaques, and thickened dermis. Usually, the ear lobes and
extremities are affected. Diffuse lepromatosis is seen usually in patients from Mexico,
who show areas of diffuse dermal infiltration and no focal lesions. Untreated,
lepromatous leprosy results in a high level of continuous bacteremia. Peripheral
neuropathy is symmetric and generalized. There is a characteristic deformity
associated with this type of leprosy, "saddle-nose deformity," which occurs because of
infiltration of the upper respiratory system and nasal cartilage. Other upper respiratory
system effects include chronic nasal congestion and epistaxis. The tuberculoid, or
paucibacillary, form of leprosy is characterized by one or few hypopigmented macules,
which are anesthetic and variable in size. These macules have distinct and elevated
borders. Peripheral neuropathy in this form of leprosy is usually asymmetric and affects
large nerves. Neural leprosy is characterized by functional impairment of large nerve
trunks without skin lesions. The upper respiratory system is not involved. M leprae
Infection. Leprosy generally requires a long duration of treatment, and compliance is a
major problem. Lepromatous leprosy requires a longer treatment time course than
tuberculoid leprosy because of the greater number of organisms involved. Currently,
the recommended treatment is dapsone and rifampin for 6 months for
tuberculoid leprosy. Dapsone alone is not recommended because of reports of
emerging resistance. For lepromatous leprosy, dapsone with rifampin or clofazimine for
24 months is recommended. Nonetheless, there have been reports of relapses even
after such long courses of treatment. Other agents such as ethionamide,
prothionamide, the aminoglycosides, minocycline, clarithromycin, and the
fluoroquinolones may also prove to be beneficial.

1092.- El tratamiento de elección para la infección gonococcica no complicada


del cervix y la uretra es:

a) Ciprofloxacina
b) TMP-SMX
c) Ceftriaxona
d) Amoxicilina
e) Neomicina

Transmission associated with unprotected sex. Purulent urethral discharge, dysuria in


men. Vaginal discharge, dysuria, intermenstrual bleeding in women. Dermatitis-arthritis

MX-Biomedical Research Group / JD-MD Bioinformatics Labs 56


JD-MD Medicine and Technology
ENARM 11ª PARTE by JD-MD

syndrome with disseminated infection. Gram-negative diplococci inside neutrophils on


stained smears of urethral, cervical, skin, or joint material. Fastidious organism
requires special media and growth conditions. DNA amplification methods (PCR, LCR)
offer improved sensitivity and specificity. Genital infection with N gonorrhoeae most
often presents as urethritis in men and cervicitis in women. Ceftriaxone in a single
dose is the treatment of choice for uncomplicated gonorrhea at all sites.
Ciprofloxacin has the advantages of a single oral dose, less expense than ceftriaxone,
and effectiveness in patients who are intolerant of cephalosporins; however, resistance
has been observed with ciprofloxacin, and it is not active against Treponema pallidum.
Complicated infections or disseminated gonococcal infections, such as arthritis,
perihepatitis, or bacteremia resulting in petecchial or pustular skin lesions, require a
higher dose of antibiotics than uncomplicated gonorrhea infections and a longer
duration of therapy. These regimens should be continued for 24-48 h after clinical
improvement is noted (ie, they become afebrile, or joint erythema or skin lesions
improve) and then switched to an oral regimen for a total of 7 d. Patients with
gonococcal meningitis or endocarditis require higher doses and longer duration of
therapy. Meningitis is treated with ceftriaxone for 14 d. Endocarditis should be treated
for = 4 wk. Concurrent infection with C trachomatis is estimated to occur in 40% of
those infected with N gonorrhoeae. Therefore doxycycline or azithromycin must be
added to the antigonococcal regimen. In pregnant women, for whom doxycycline is
contraindicated, an erythromycin base can be used. If erythromycin cannot be
tolerated, amoxicillin for 7-10 d is indicated. Antibiotic resistance has been a major
concern, as penicillin resistance has spread and, more recently, fluoroquinolone
resistance has been increasingly reported. Beta-lactams remain the drugs of choice in
most instances. Antibiotic resistance occurs by one or more mechanisms, including
chromosomal mutations leading to decreased penicillin-binding-protein affinity,
decreased outer membrane permeability, or beta-lactamase production.

1093.- Paciente de 35 años, 12 horas después de la ingesta de alimentos mal


preparados presenta nausea, vomitos y dolor abdominal. Al examen físico se
encuentra ptosis bilateral, midriasis con reacción lenta y paralisis flacida
aguda. Entre las medidas urgentes que se deben efectuar para tratar a este
paciente, la primordial consiste en:

a) Antitoxina botulinica por vía intravenosa


b) Intubación orotraqueal y oxigeno al 100%
c) Traqueostomia electiva temprana
d) Antiespasmodicos y antihelminticos
e) Dosis altas de penicilina sodica por via intravenosa

This clinical syndrome is caused by the neurotoxin of C botulinum. There are seven
types (A-G) of neurotoxin, all of which inhibit the release of acetylcholine at the level of
peripheral neuromuscular junctions. In the majority of cases, the disease is acquired by
ingestion of preformed toxin in home-canned vegetables, fruits, and fish. In Japan, the
former Soviet Union, Scandinavia, and the Great Lakes region of the United States,
type E toxin causes disease in people who consume raw or lightly smoked fish. There
are four categories of botulism: Food borne (most common), Wound botulism caused
by the absorption of toxin from a wound contaminated by C botulinum (the rarest
form), Infant botulism resulting from in vivo elaboration of toxin by colonizing
organisms in the bowel, Undetermined, which refers to cases that occur in individuals
> 1 year old in whom no food or wound source is identified. Symptoms and signs arise
12-36 h after food ingestion and consist of acute onset and progressive flaccid
paralysis involving the facial musculature and the cranial nerves bilaterally, then
descending symmetrically to the pharynx, thoracic region, and the upper and lower
extremities. This evolves into respiratory failure without impairment of consciousness.

MX-Biomedical Research Group / JD-MD Bioinformatics Labs 57


JD-MD Medicine and Technology
ENARM 11ª PARTE by JD-MD

Fever is classically absent. In cases of infant botulism, which is found in children 6 days
to 11 months old, constipation is the initial symptom, followed by lethargy, feeding
difficulties, altered cry, floppiness, ophthalmoplegia, and respiratory failure. Treatment
is mainly supportive; antitoxin made from equine serum can be used. In the United
States, it is obtained through state health departments or the Centers for Disease
Control. The standard dose is one vial intravenously and one vial intermittently. It may
be repeated every 4 h in severe progressive cases. Clinical trials evaluating its efficacy
are lacking. Full recovery takes from 3 months to 1 year. Risk of death ranges from 4 to
25%, depending on the promptness with which the diagnosis is made.

1094.- Germen causal frecuente de la otitis media aguda:

a) Pseudomona sp.
b) Streptococcus pneumoniae
c) Streptococcus pyogenes
d) Klebsiella sp.
e) Haemophylis influenzae

Acute otitis media is a bacterial infection of the mucosally lined air-containing spaces of
the temporal bone. Purulent material forms not only within the middle ear cleft but also
within the mastoid air cells and petrous apex when they are pneumatized. Acute otitis
media is usually precipitated by a viral upper respiratory tract infection that causes
auditory tube edema. This results in accumulation of fluid and mucus, which becomes
secondarily infected by bacteria. The most common pathogens both in adults and
in children are Streptococcus pneumoniae, Haemophilus influenzae, and
Streptococcus pyogenes. Acute otitis media is most common in infants and children,
although it may occur at any age. Presenting symptoms and signs include otalgia, aural
pressure, decreased hearing, and often fever. The typical physical findings are
erythema and decreased mobility of the tympanic membrane. Occasionally, bullae will
be seen on the tympanic membrane. Although it is taught that this represents infection
with Mycoplasma pneumoniae, most cases involve more common pathogens. Rarely,
when middle ear empyema is severe, the tympanic membrane can be seen to bulge
outward. In such cases, tympanic membrane rupture is imminent. Rupture is
accompanied by a sudden decrease in pain, followed by the onset of otorrhea. With
appropriate therapy, spontaneous healing of the tympanic membrane occurs in most
cases. When perforation persists, chronic otitis media frequently evolves. Mastoid
tenderness often accompanies acute otitis media and is due to the presence of pus
within the mastoid air cells. This alone does not indicate suppurative (surgical)
mastoiditis. The treatment of acute otitis media is specific antibiotic therapy, often
combined with nasal decongestants. The first-choice antibiotic treatment is either
amoxicillin (20-40 mg/kg/d) or erythromycin (50 mg/kg/d) plus sulfonamide (150
mg/kg/d) for 10 days. Alternatives useful in resistant cases are cefaclor (20-40
mg/kg/d) or amoxicillin-clavulanate (20-40 mg/kg/d) combinations. Tympanocentesis
for bacterial (aerobic and anaerobic) and fungal culture may be performed by any
experienced physician. A 20-gauge spinal needle bent 90 degrees to the hub attached
to a 3-mL syringe is inserted through the inferior portion of the tympanic membrane.
Interposition of a pliable connecting tube between the needle and syringe permits an
assistant to aspirate without inducing movement of the needle. Tympanocentesis is
useful for otitis media in immunocompromised patients and when infection persists or
recurs despite multiple courses of antibiotics. Surgical drainage of the middle ear
(myringotomy) is reserved for patients with severe otalgia or when complications of
otitis (eg, mastoiditis, meningitis) have occurred. Recurrent acute otitis media may be
managed with long-term antibiotic prophylaxis. Single daily doses of sulfamethoxazole
(500 mg) or amoxicillin (250 or 500 mg) are given over a period of 1-3 months. Failure
of this regimen to control infection is an indication for insertion of ventilating tubes.

MX-Biomedical Research Group / JD-MD Bioinformatics Labs 58


JD-MD Medicine and Technology
ENARM 11ª PARTE by JD-MD

1095.- Para precisar la presencia de diverticulitis se debe realizar:

a) Ultrasonografia
b) Rectosigmoidoscopia
c) Tomografia computarizada
d) Colonoscopia
e) Colon por enema

Colonic diverticulosis increases with age, ranging from 5% in those under age 40, to
30% at age 60, to more than 50% over age 80 years in Western societies. In contrast, it
is very uncommon in developing countries with much lower life expectancies. Most are
asymptomatic, discovered incidentally at endoscopy or on barium enema.
Complications in one-third include lower gastrointestinal bleeding and diverticulitis.
Colonic diverticula may vary in size from a few millimeters to several centimeters and
in number from one to several dozen. Almost all patients with diverticulosis have
involvement in the sigmoid colon; however, only 15% have proximal colonic disease. In
most patients, diverticulosis is believed to arise after many years of a diet deficient in
fiber. The undistended, contracted segments of colon have higher intraluminal
pressures. Over time, the contracted colonic musculature, working against greater
pressures to move small, hard stools, develops hypertrophy, thickening, rigidity, and
fibrosis. Diverticula may develop more commonly in the sigmoid because intraluminal
pressures are highest in this region. The extent to which abnormal motility and
hereditary factors contribute to diverticular disease is unknown. Patients with diffuse
diverticulosis may have an inherent weakness in the colonic wall. Patients with
abnormal connective tissue are also disposed to development of diverticulosis,
including Ehlers-Danlos syndrome, Marfan's syndrome, and scleroderma. Perforation of
a colonic diverticulum results in an intra-abdominal infection that may vary from
microperforation (most common) with localized paracolic inflammation to
macroperforation with either abscess or generalized peritonitis. Thus, there is a range
from mild to severe disease. Most patients with localized inflammation or infection
report mild to moderate aching abdominal pain, usually in the left lower quadrant.
Constipation or loose stools may be present. Nausea and vomiting are frequent. In
many cases, symptoms are so mild that the patient may not seek medical attention
until several days after onset. Physical findings include a low-grade fever, left lower
quadrant tenderness, and a palpable mass. Stool occult blood is common, but
hematochezia is rare. Leukocytosis is mild to moderate. Patients with free perforation
present with a more dramatic picture of generalized abdominal pain and peritoneal
signs. Plain abdominal films are obtained in all patients to look for evidence of free
abdominal air (signifying free perforation), ileus, and small or large bowel obstruction.
In patients with mild symptoms and a presumptive diagnosis of diverticulitis, empiric
medical therapy is started without further imaging in the acute phase. Patients who
respond to acute medical management should undergo complete colonic evaluation
with colonoscopy or barium enema after resolution of clinical symptoms to corroborate
the diagnosis or exclude other disorders such as colonic neoplasms. In patients who do
not improve rapidly after 2-4 days of empiric therapy and in those with severe disease,
CT scan of the abdomen is obtained to look for evidence of diverticulitis,
including colonic diverticula and wall thickening, pericolic fat infiltration, abscess
formation, or extraluminal air or contrast. Endoscopy and barium enema are
contraindicated during the initial stages of an acute attack because of the risk of free
perforation, though sigmoidoscopy with minimal air insufflation is sometimes required
to exclude other diagnoses. Most patients can be managed with conservative
measures. Patients with mild symptoms and no peritoneal signs may be managed
initially as outpatients on a clear liquid diet and broad-spectrum oral antibiotics with
anaerobic activity. Reasonable regimens include amoxicillin and clavulanate potassium

MX-Biomedical Research Group / JD-MD Bioinformatics Labs 59


JD-MD Medicine and Technology
ENARM 11ª PARTE by JD-MD

(875 mg/125 mg) twice daily; or metronidazole, 500 mg three times daily; plus either
ciprofloxacin, 500 mg twice daily, or trimethoprim-sulfamethoxazole, 160/800 mg twice
daily orally, for 7-10 days or until the patient is afebrile for 3-5 days. Symptomatic
improvement usually occurs within 3 days, at which time the diet may be advanced.
Patients with increasing pain, fever, or inability to tolerate oral fluids require
hospitalization. Patients with severe diverticulitis (high fevers, leukocytosis, or
peritoneal signs) and patients who are elderly or immunosuppressed or who have
serious comorbid disease require hospitalization acutely. Patients should be given
nothing by mouth and should receive intravenous fluids. If ileus is present, a
nasogastric tube should be placed. Intravenous antibiotics should be given to cover
anaerobic and gram-negative bacteria. Single-agent therapy with either a second-
generation cephalosporin (eg, cefoxitin), piperacillin-tazobactam, or ticarcillin
clavulanate appears to be as effective as combination therapy (eg, metronidazole or
clindamycin plus an aminoglycoside or third-generation cephalosporin [eg, ceftazidime,
cefotaxime]). Symptomatic improvement should be evident within 2-3 days. The
antibiotics should be continued for 7-10 days, after which time elective evaluation with
colonoscopy or barium enema should be performed. Approximately 20-30% of patients
with diverticulitis will require surgical management. Surgical consultation should be
obtained on all patients with severe disease or those who fail to improve after 72 hours
of medical management. Indications for emergent surgical management include free
peritonitis and large abscesses. Patients with fistulas or colonic obstruction due to
chronic disease will require elective surgery. Patients with a localized abdominal
abscess can be treated acutely with a percutaneous catheter drain placed by an
interventional radiologist. This permits control of the infection and resolution of the
immediate infectious inflammatory process. In this manner, a subsequent single-stage
elective surgical operation can be performed in which the diseased segment of colon is
removed and primary colonic anastomosis performed. In patients in whom catheter
drainage is not possible or helpful or in cases requiring emergency surgery, it is
necessary to perform surgery in two stages. In the first stage, the diseased colon is
resected and the proximal colon brought out to form a temporary colostomy. The distal
colonic stump is either closed (forming a Hartmann pouch) or exteriorized as a mucous
fistula. Weeks later, after inflammation and infection have completely subsided, the
colon can be reconnected electively.

1096.- El tratamiento farmacologico complementario en caso de cólera en el


adulto es:

a) Trimetoprim con sulfametoxazol


b) Ciproxina
c) Ceftrixona
d) Rifampicina
e) Doxiciclina

Cholera is a fulminant diarrheal disease caused by V cholerae. Patients with cholera


develop frequent, watery stools, which may reach volumes =1 L/h. If these patients are
untreated, they will develop rapid dehydration and electrolyte abnormalities, which
may result in death within hours from the onset of disease. Therefore cholera is a
medical emergency. V cholerae is noninvasive and produces diarrhea by the
elaboration of a potent enterotoxin. This enterotoxin consists of two subunits that have
been well characterized. The circular, pentameric B-subunit binds monosialosyl
ganglioside residues on the surface of the intestinal epithelial cell and introduces the
enzymatic A-subunit into the cytoplasm. The A-subunit functions as an adenosine
diphosphate (ADP) ribosylase and transfers an ADP-ribose moiety from cytoplasmic
nicotinamide adenine dinucleotide (NAD) to the cell-membrane-associated adenylate
cyclase. ADP-ribosylation renders adenylate cyclase unresponsive to feedback

MX-Biomedical Research Group / JD-MD Bioinformatics Labs 60


JD-MD Medicine and Technology
ENARM 11ª PARTE by JD-MD

inhibition. The result is dramatically increased intracellular cyclic adenosine


monophosphate (cAMP) levels. Elevated cytoplasmic cAMP levels cause massive fluid
and electrolyte transit into the bowel lumen, resulting in the characteristic "rice water"
stool of cholera. Patients with cholera initially develop increased peristalsis, a feeling of
fullness, and occasionally vomiting. Diarrhea develops rapidly and soon becomes
watery. Abdominal pain and high fever are usually minimal or absent and distinctly
disproportionate to the amount of diarrhea. Fever, when present, is usually caused by
dehydration. Electrolyte abnormalities, from voluminous diarrhea, may manifest as
muscle weakness, intestinal ileus, or even cardiac dysrhythmia. The clinical
manifestations of infection and the possibility of severe complications correlate with
the patient's hydration and electrolyte abnormalities. Mental status changes secondary
to hypoglycemia may also occur. Without urgent fluid and electrolyte replacement
therapy, hypovolemic shock and death may occur rapidly. Primary therapy of cholera
consists of fluid and electrolyte replacement. Published guidelines for fluid and
electrolyte replacement in cholera are available. Either a citrate (10 mmol/L)- or
bicarbonate (30 mmol/L)-based solution may be used for oral rehydration. These should
contain sodium (90 mmol/L), potassium (20 mmol/L), chloride (80 mmol/L), and glucose
(111 mmol/L). The glucose is actively transported into enterocytes. This osmotically
drives water from the lumen and into the body. In developing countries, the water
residua from boiled rice may be used for rehydration. This is an inexpensive and
excellent source of oligosaccharides, which like glucose, aid in the osmotic shifting of
water from the lumen to the body. Intravenous fluid replacement therapy is necessary
for patients with severe dehydration (=10% of their body weight) and acidosis (pH
<7.2). These patients should be rehydrated with 50% normal saline with 44 mmol/L of
bicarbonate. Potassium replacement may be given as needed. If vomiting is not
prominent, oral replacement is usually adequate. If mental status changes are present,
hypoglycemia should be treated with intravenous glucose infusion (bolus 3-4 mL/kg of
a 25% glucose solution, followed by continuous infusion of 10 mg/kg/h).

Treatment: Fluid and Electrolyte Replacement, Acid-Base, and Glucose


Management Principal Consideration
First Choice
Children: Ampicillin, 250 mg orally or IV every 6 h for 5 d
Adults: Tetracycline, 250 mg orally or IV every 6 h for 5 d
During pregnancy, ampicillin is the drug of choice
Second Choice
Children: TMP/SMX orally or IV (>2 months old); TMP, 3-6 mg, + SMX, 15-30 mg/kg
every 12 h for 5 d
Adults:Ampicillin, 250 mg orally or IV every 6 h for 5 d
Penicillin Allergic
Children: TMP/SMX OR Furazolidone (liquid = 3.33 mg/ml): >5 years old, 7.5-15 ml;
1-4 years old, 5.0-7.5 ml; 1 month-1 year old, 2.5-5.0 ml orally every 6 h for 5 d
Adults:Tetracycline OR TMP/SMX2 orally or IV: TMP, 160 mg, + SMX, 800 mg orally or IV
every 12 h for 5 d OR Furazolidone, 100 mg orally every 6 h for 5 d

1097.- Paciente masculino de 17 años de edad que se presenta con


sudoración, palidez, hipertensión arterial, nausea y vomito. El diagnóstico
más probable es:

a) Glomerulonefritis
b) Feocromocitoma
c) Síndrome nefrótico
d) Insuficiencia suprarrenal
e) Tumor renal

MX-Biomedical Research Group / JD-MD Bioinformatics Labs 61


JD-MD Medicine and Technology
ENARM 11ª PARTE by JD-MD

Pheochromocytomas are rare, being found in less than 0.3% of hypertensive


individuals. The incidence is higher in patients with moderate to severe hypertension.
Pheochromocytomas can be lethal unless they are diagnosed and treated
appropriately. They typically cause attacks of severe headache (80% of patients),
perspiration (70% of patients), and palpitations (60% of patients); other symptoms
may include anxiety (50% of patients), a sense of impending doom, or tremor (40% of
patients). Vasomotor changes during an attack cause mottled cyanosis and facial
pallor; as the attack subsides, facial flushing may occur as a result of reflex
vasodilation. Other findings may include tachycardia, precordial or abdominal pain,
vomiting, increasing nervousness and irritability, increased appetite, and loss of
weight. Anginal attacks may occur. Physical findings usually include
hypertension (90% of patients), which may be sustained (20% of patients),
sustained with paroxysms (50% of patients), or paroxysmal only (25% of patients).
There may be cardiac enlargement and cardiomyopathy, postural tachycardia (change
of more than 20 beats/min) and postural hypotension, and mild elevation of basal body
temperature. Retinal hemorrhage or cerebrovascular hemorrhage occurs occasionally.
Catastrophic hypertensive crisis and fatal cardiac arrhythmias can occur spontaneously
or may be triggered by intravenous contrast dye or glucagon injection, needle biopsy of
the mass, anesthesia, and surgical procedures. The manifestations of
pheochromocytoma are quite varied and mimic other conditions. Some patients are
normotensive and asymptomatic. In addition to the above symptoms, some patients
can present with psychosis or confusion, seizures, hyperglycemia, bradycardia,
hypotension, constipation, paresthesias, or Raynaud's phenomenon. Other patients
may have pulmonary edema and heart failure due to cardiomyopathy. Epinephrine
secretion may cause episodic tachyarrhythmias, hypotension, or syncope. Some
patients may be entirely asymptomatic despite high serum levels of catecholamines.
Others may present with abdominal discomfort from a large hemorrhagic
pheochromocytoma, or with pain from metastatic disease. In addition to
catecholamines and their metabolites, pheochromocytomas secrete a wide range of
other peptides that can sometimes cause symptoms of Cushing's syndrome (ACTH),
erythrocytosis (erythropoietin), or hypercalcemia (PTHrP). Hypermetabolism is present;
thyroid function tests are normal, including serum T4, FT4, T3, and TSH. Hyperglycemia
is present in about 35% of patients but is usually mild. Leukocytosis is common. The
ESR is sometimes elevated. Plasma renin activity may be increased by catecholamines.
Assay of urinary catecholamines and metanephrines (total and fractionated) and
creatinine detects most pheochromocytomas, especially when samples are obtained
during or immediately following an episodic attack. A 24-hour urine specimen is usually
obtained, although an overnight or shorter collection may be used; patients with
pheochromocytomas generally have more that 2.2 mcg of total metanephrine per
milligram of creatinine, and more than 135 mcg total catecholamines per gram
creatinine. Urinary assay for total metanephrines is about 97% sensitive for detecting
functioning pheochromocytomas. Urinary assay for vanillylmandelic acid (VMA) is about
89% sensitive and is not usually required.

1098.- El agente que con mayor frecuencia produce el síndrome


coqueluchoide es:

a) Streptococo beta-hemolítico
b) Diplococo Pneumoniae
c) Haemophylus influenzae
d) Mycoplasma pneumoniae
e) Bordetella pertussis

Classical pertussis occurs in three clinical stages: catarrhal, paroxysmal, and


convalescent. The catarrhal stage is characterized by nonspecific upper respiratory

MX-Biomedical Research Group / JD-MD Bioinformatics Labs 62


JD-MD Medicine and Technology
ENARM 11ª PARTE by JD-MD

symptoms, including rhinorrhea, mild cough, and low-grade fever. During this stage,
which typically lasts 1-2 weeks, the disease is highly communicable. The paroxysmal
stage is marked by sudden attacks or paroxysms of severe, repetitive coughing, often
culminating with the characteristic whoop (tos coqueluchoide-whooping cough-
Tos espasmódica en la que la primera inspiración ocurre con la glotis
semicerrada, produciendo un ruido característico) and frequently followed by
vomiting. A marked lymphocytosis usually accompanies this stage of the disease, with
lymphocyte counts sometimes exceeding 50,000/mm3 and usually representing 70%
or more of total circulating leukocytes. The paroxysmal stage typically lasts 1-4 weeks
and can be associated with a variety of complications, including secondary bacterial
infections such as pneumonia and otitis media, toxic central nervous system
manifestations such as seizures and encephalopathy, and effects of increased
intrathoracic and intra-abdominal pressure such as pneumothorax, hernia, and rectal
prolapse. The beginning of the convalescent (recovery) stage is marked by a reduction
in frequency and intensity of coughing spells. After clinical pertussis, immunity to
disease is lifelong. Although most cases of pertussis follow a characteristic course,
exceptions exist. In infants < 3 months, the catarrhal stage is usually no longer than a
few days, and the paroxysmal and convalescent stages are extremely protracted, with
coughing spells that continue throughout the first year of life. In infants < 6 months,
apnea is a common manifestation, and the whoop is often absent. Paradoxically, in
infants, cough and whoop may become louder and more classic during convalescence,
reflecting growth in body mass and increased strength. In immunized children, all three
stages are shortened, and in adults, only a protracted cough may be present. Post-
tussive vomiting is common in pertussis at all ages and is a major clue to the diagnosis
in adolescents and adults. With subsequent respiratory illnesses over the next several
months, paroxysmal coughing may recur, though not because of recurrence of active
Bordetella infection.

1099.- La medida más adecuada para iniciar el tratamiento de la diverticulitis


consiste en usar:

a) Dieta blanda con bajo residuo


b) Dieta blanda con alto residuo
c) Metronidazol y cefalexina
d) Ciprofloxacino y clindamicina
e) Cefotaxima y metronidazol

Most patients can be managed with conservative measures. Patients with mild
symptoms and no peritoneal signs may be managed initially as outpatients on a clear
liquid diet and broad-spectrum oral antibiotics with anaerobic activity. Reasonable
regimens include amoxicillin and clavulanate potassium (875 mg/125 mg) twice daily;
or metronidazole, 500 mg three times daily; plus either ciprofloxacin, 500 mg twice
daily, or trimethoprim-sulfamethoxazole, 160/800 mg twice daily orally, for 7-10 days
or until the patient is afebrile for 3-5 days. Symptomatic improvement usually occurs
within 3 days, at which time the diet may be advanced. Patients with increasing pain,
fever, or inability to tolerate oral fluids require hospitalization. Patients with severe
diverticulitis (high fevers, leukocytosis, or peritoneal signs) and patients who are
elderly or immunosuppressed or who have serious comorbid disease require
hospitalization acutely. Patients should be given nothing by mouth and should receive
intravenous fluids. If ileus is present, a nasogastric tube should be placed. Intravenous
antibiotics should be given to cover anaerobic and gram-negative bacteria. Single-
agent therapy with either a second-generation cephalosporin (eg, cefoxitin),
piperacillin-tazobactam, or ticarcillin clavulanate appears to be as effective as
combination therapy (eg, metronidazole or clindamycin plus an aminoglycoside or
third-generation cephalosporin [eg, ceftazidime, cefotaxime]). Symptomatic

MX-Biomedical Research Group / JD-MD Bioinformatics Labs 63


JD-MD Medicine and Technology
ENARM 11ª PARTE by JD-MD

improvement should be evident within 2-3 days. The antibiotics should be continued for
7-10 days, after which time elective evaluation with colonoscopy or barium enema
should be performed. Approximately 20-30% of patients with diverticulitis will require
surgical management. Surgical consultation should be obtained on all patients with
severe disease or those who fail to improve after 72 hours of medical management.
Indications for emergent surgical management include free peritonitis and large
abscesses. Patients with fistulas or colonic obstruction due to chronic disease will
require elective surgery. Patients with a localized abdominal abscess can be treated
acutely with a percutaneous catheter drain placed by an interventional radiologist. This
permits control of the infection and resolution of the immediate infectious inflammatory
process. In this manner, a subsequent single-stage elective surgical operation can be
performed in which the diseased segment of colon is removed and primary colonic
anastomosis performed. In patients in whom catheter drainage is not possible or
helpful or in cases requiring emergency surgery, it is necessary to perform surgery in
two stages. In the first stage, the diseased colon is resected and the proximal colon
brought out to form a temporary colostomy. The distal colonic stump is either closed
(forming a Hartmann pouch) or exteriorized as a mucous fistula. Weeks later, after
inflammation and infection have completely subsided, the colon can be reconnected
electively.

1100.- El cuadro característico que comprende ataque agudo bilateral de


pares craneales, debilidad con parálisis de vías descendentes, transtornos de
la visión, disfagia, boca seca, parálisis flácida simétrica, sin fiebre,
relacionada al consumo de alimentos contaminados corresponde a:

a) Cólera
b) Shigelosis
c) Giardiasis
d) Botulismo
e) Tétanos

Symptoms and signs fo botilusm arise 12-36 h after food ingestion and consist of acute
onset and progressive flaccid paralysis involving the facial musculature and the cranial
nerves bilaterally, then descending symmetrically to the pharynx, thoracic region, and
the upper and lower extremities. This evolves into respiratory failure without
impairment of consciousness. Fever is classically absent. In cases of infant botulism,
which is found in children 6 days to 11 months old, constipation is the initial symptom,
followed by lethargy, feeding difficulties, altered cry, floppiness, ophthalmoplegia, and
respiratory failure.

MX-Biomedical Research Group / JD-MD Bioinformatics Labs 64


JD-MD Medicine and Technology

You might also like